110 mid term

Ace your homework & exams now with Quizwiz!

A mother is bringing her infant into the the clinic for a well baby check. The infant's weight gain is on target for age. A correctly written evaluative statement is which of the following?

"8/2/2014. Goal met. The infant's weight gain is appropriate for age."

The nurse is discussing car safety with the mother of a 6-year-old child. The child's mother questions the need for the use of special car seats for her child. What information can be provided to her?

"At the age of 6 your child should be using a booster seat."

A client has just been given a diagnosis of cirrhosis of the liver. Which statements by the nurse should be avoided because they could impede communication? Select all that apply.

"Cheer up. Tomorrow is another day." "Your doctor knows best." "Don't worry. You will be just fine in another day or two." "Everything will be all right."

A nurse is conducting a health history for a patient with a chronic respiratory problem. What question might the nurse ask to assess for orthopnea? A) "Do you have problems breathing when you walk up stairs?" B) "Does your medication help you breathe better?" C) "How many pillows do you sleep on at night to breathe better?" D) "Tell me about your breathing difficulties since you stopped smoking."

"How many pillows do you sleep on at night to breathe better?"

Which of the following entires would be an example of appropriate documentation? A. Patient appears depressed and tired B. "I am so down today, and i just don't have any energy." C. Patient had a good bowel movement D. Complains of abdominal pain, Probably constipated

"I am so down today, and i just don't have any energy."

A nurse is asking a colleague about a situation. Which statement demonstrates assertive communication?

"I think there is a better way to handle this."

The nurse has just measured an adult client's oral temperature and obtained a result of 102.4ºF (39.1ºC). The client states, "I just finished my coffee right before you came in. Can I have another cup?" Which response by the nurse is most appropriate?

"I will bring you another cup when I return in 30 minutes to reassess your temperature. Please do not drink any other beverages until I return."

The poison control nurse receives a call from the parent of a 2-year-old child. The parent states, "I just took a quick shower, and when I finished, I walked into the kitchen and found my child with an open bottle of household cleaner." What is the poison control nurse's appropriate response?

"Is your child breathing at this time?"

A nurse is explaining a chest tube to family members who do not understand where it is placed. What would the nurse tell them?

"It is inserted into the space between the lining of the lungs and the ribs."

A home healthcare nurse notices that his assigned patient uses a mercury thermometer. He asks the nurse what to do if it breaks. Which of the following is not correct? A) "Just flush the glass and mercury down the toilet." B) "Do not vacuum the area where it breaks." C) "Open the windows and close off the room for an hour." D) "Throw away any clothing exposed to the mercury."

"Just flush the glass and mercury down the toilet."

The nurse is preparing discharge teaching for a client with a history of recurrent pneumonia. What deep breathing techniques will the nurse plan to teach?

"Take in as much air as possible, hold your breath briefly, and exhale slowly."

The nurse at the neighborhood family clinic is instructing a 55-year-old client with hypertension and a family history of heart disease about reduction of risk factors. It is most important for the nurse to make which statement to the client?

"Take your blood pressure medications exactly as your doctor prescribed them."

A nurse pays a house visit to a client who is on parenteral nutrition (PN). The client expresses that he misses enjoying food with his family. What is the most appropriate response by the nurse?

"Tell me more about how it feels to eat with your family."

Which of the following questions or statements would be appropriate in eliciting further information when conducting a health history interview? A. "Why didn't you go the doctor when you began to have this pain?" B. "Are you feeling better now than you did during the night?" C. "Tell me more about what caused your pain." D. "If i were you, i would not wait to get medical help next time."

"Tell me more about what caused your pain."

Which statement indicates that a family understands the teaching that has been provided by the nurse related to car seat safety for a 9-month-old infant?

"We place our baby in a rear-facing car seat in the back seat of the car."

A client comes to the clinic reporting abdominal pain. Which question would be most appropriate for the nurse to ask to facilitate the assessment?

"What activities exaggerate the pain?"

The client is under immediate stress. The nurse assesses which sign as an effect of the sympathetic system? - A) Blood sugar of 65 mg/dL - B) Heart rate of 102 beats/minute - C) Increased bowel sounds - D) Cool, clammy skin

- Heart rate of 102 beats/minute

Of the following physiologic stressors, which one is a physical agent? - A) Heat - B) Drugs - C)Bacteria - D) Hypoxia

- Heat

A woman who was assaulted in the street is brought to the emergency room for observation. A nurse documents that the woman has difficulty communicating verbally, is agitated, and complains of chest pain and a sense of impending doom. What type of anxiety is this client experiencing? - A) Mild anxiety - B) Moderate anxiety - C) Severe anxiety - D) Panic

- Panic

While reading a physician's progress notes, a student notes that an assigned patient is having hypoxia. What abnormal assessments would the student expect to find?

-Dyspnea -Tachycardia -Cyanosis

A nurse researching a diet for a client with diabetes includes foods that supply energy to the body. Which of the following are classes of nutrients that supply this energy? Select all that apply.

-Proteins -Fats -Carbohydrates

When creating the teaching plan for a client that will be monitoring his or her pulse at home, which factors should the nurse teach the client that may influence the pulse rate by causing an increase in pulse? Select all that apply.

-Stress -Fever -Exercise

A

14. Which of the following is categorized as a psychomotor outcome? A) Within 2 days of teaching, the patient's wife will demonstrate abdominal dressing change. B) Within 1 week of attending class, the patient will have cut smoking from 20 to 10 cigarettes per day. C) The patient will verbalize understanding of need to continue to take medications as prescribed. D) The patient's skin will remain smooth, moist, and without breakdown or ulceration.

B

27. Which of the following is an example of a well-stated nursing intervention? A) Patient will drink 100 mL of water every 2 hours while awake. B) Offer patient 100 mL of water every 2 hours while awake. C) Offer patient water when he complains of thirst. D) Patient will continue to increase oral intake when awake.

B

6. A nurse is discharging a patient from the hospital. When should discharge planning be initiated? A) at the time of discharge from an acute healthcare setting B) at the time of admission to an acute healthcare setting C) before admission to an acute healthcare setting D) when the patient is at home after acute care

D

8. A father runs into the emergency room with his 18-month-old son in his arms. The father screams, Help, he is not breathing! The nursing diagnosis of Impaired Gas Exchange is what level of priority diagnosis? A) no priority B) low priority C) medium priority D) high priority

A

9. The nursing diagnosis Impaired Gas Exchange, prioritized by Maslow's hierarchy of basic human needs, is appropriate for what level of needs? A) physiologic B) safety C) love and belonging D) self-actualization

The nurse is conducting the physical assessment of a client at the health care facility. The nurse uses the pulse oximetry technique to monitor the oxygen saturation in the client's blood. Which of the following pulse oximeter ranges indicates that the client is adequately oxygenated?

95%-100%

The nurse would expect to recommend an oxygen tent fro which of the following patients?

A child who will not leave a facemask or cannula in place

Which individual is at greater risk for respiratory illnesses from environmental causes?

A factory worker in a large city

A nurse caring for a client admitted with a deep vein thrombosis is individualizing a prepared plan of care that identifies nursing diagnoses, outcomes, and related nursing interventions common to this condition. What type of tool is the nurse using?

A standardized care plan

A nurse is preparing a clinical outcome for a 32-year-old female runner who is recovering from a stroke that caused right-sided paresis. An example of this type of outcome is: a. After receiving 3 weeks of physical therapy, patient will demonstrate improved movement on the right side of her body. b. By 8/15/15, patient will be able to use right arm to dress, comb hair, and feed herself. c. Following physical therapy, patient will begin to gradually participate in walking/running events. d. By 8/15/15, patient will verbalize feeling sufficiently prepared to participate in running events.

A. After receiving 3 weeks of physical therapy, patient will demonstrate improved movement on the right side of her body.

A nurse is using a structured care methodology that follows a set of steps based on a clinician's decision process to help standardize nursing care plans. What is the term for this element of a structured care methodology? A) algorithm B) national guidelines C) standard of care D) clinical practice guideline

A. algorithm

Which expected client outcome is an example of a psychomotor outcome? Select all that apply.

Accurately drawing up insulin. Safely ambulating using a walker.

A client has been diagnosed with pneumonia and is experiencing chest pain when taking a deep breath. What are the two priority nursing diagnoses?

Acute Pain Ineffective Airway Clearance

The nurse is collecting data on a client presenting to the medical short-stay unit for a colonoscopy. A client reports to the nurse that he quit smoking six months ago after being diagnosed with lung cancer. The nurse recognizes this change in behavior is which type of outcome?

Affective outcome

A nurse is developing outcomes for a specific problem statement. What is one of the most important considerations the nurse should have? A) that the written outcomes are designed to meet nursing goals B) to encourage the patient and family to be involved C) to discourage additions by other healthcare providers D) why the nurse believes the outcome is important

B.

A nurse is discussing infant care with a woman who just had a baby girl. What type of nutrition would the nurse recommend for the infant?

Breast-feeding or formula with iron

What category of medications may be administered by nebulizer or metered-dose inhaler to open narrowed airways?

Bronchodilators

What is one reason for the "middle-aged spread" often seen in middle adults? A) changes in hormones B) loss of satisfactory roles C) decreased physical activity D) satisfaction with one's life

C "Middle age spread is a term commonly used to describe the accumulation of fat around the hips, bottom and belly in men and women as they get older and is often associated with decreasing core body strength,"

A nurse writes down the following outcome for a depressed patient: By 6/9/12, the patient will state three positive benefits of receiving counseling. This is an example of which of the following types of outcomes: A) psychomotor B) cognitive C) affective D) realistic

C. affective

What part of the nursing diagnosis statement suggests the nursing interventions to be included in the plan of care? A) problem statement B) defining characteristics C) etiology of the problem D) outcomes criteria

C. etiology of the problem

A nurse assesses the vital signs of a patient who is one day post-surgery in which a colostomy was performed. The nurse then uses the data to update the patient plan of care. What are these actions considered? A) initial planning B) comprehensive planning C) on-going planning D) discharge planning

C. on-going planning

From what part of the nursing diagnoses are outcomes derived during outcome identification and planning? A) the defining characteristics B) the related factors C) the problem statement D) the database

C. the problem statement

What guides professional practice?

CNA Standards of Nursing Practice

A nurse is caring for a client with excessive abdominal fat. Which of the following is a risk associated with excessive abdominal fat about which the nurse should inform the client?

Cardiovascular disease

You are the nurse who is working in the prenatal clinic. Today you are seeing Mrs. Evans, who expects her baby in the next week. Mrs. Evans asks what the hardest adjustment for her baby will be at birth. The nurse correctly replies with which of the following answers?

Changes in the respiratory system

A pediatric nurse practitioner is assessing the development of a child. The nurse's assessment of development will focus on which of the following?

Changes in thoughts, feelings, and behaviors

A patient scheduled for a colonoscopy is scheduled to receive a hypertonic enema prior to the procedure. A hypertonic enema is classified as which type of enema? a) Cleansing enema b) Retention enema c) Carminative enema d) Return-flow enema

Cleansing enema

A nurse admitting a new client to the hospital needs to determine the client's needs and current problems. What is the priority action of the nurse?

Complete an assessment.

Which intervention performed by the nurse is most appropriate for assisting a client in meeting physiologic needs based on Maslow's Hierarchy of Needs?

Cutting up food and opening drink containers for the client.

In general, what is the focus of care for nurses who work with older adults? A) providing all necessary physical care B) referring patients for needed emotional support C) establishing goals and expected outcomes for the patient D) assisting patients to function as independently as possible

D

A nurse is caring for a patient who is diagnosed with congestive heart failure. Which statement below is not an example of a well-stated nursing intervention? a. Offer patient 60 mL of water or juice (prefers orange or cranberry juice) every 2 hours while awake for a total minimum PO intake of 500 mL. b. Teach patient the necessity of carefully monitoring fluid intake and output; remind patient each shift to mark off fluid intake on record at bedside. c. Walk with patient to bathroom for toileting every 2 hours (on even hours) while patient is awake. d. Manage patient's pain.

D. Manage patient's pain.

Although each care plan is individualized, there are certain risks and health problems that, for example, patients undergoing similar medical or surgical treatment have in common. What name is given to this type of care plan? A) initial B) ongoing C) discharge D) standardized

D. standardized

Which of the following would a nurse know is a part of an evaluative statement? Select all that apply.

Description of how the patient outcome was met Patient data that supports how the outcome was met

To promote health of the fetus, the nurse should instruct the woman in the first trimester of pregnancy to do which of the following?

Eat foods high in folic acid

Which dietary guideline would be appropriate for the older adult homebound client with advanced respiratory disease who informs the nurse that she has no energy to eat?

Eat smaller meals that are high in protein

What independent nursing intervention can be implemented to stimulate appetite?

Encourage or provide oral care.

Which phase of the nursing process enables the nurse to compare the actual outcomes with the expected outcomes? a) Evaluation b) Assessment c) Planning d) Implementation

Evaluation

Once a nurse has collected and interpreted the data on a client's outcome achievement, the nurse will then make a judgment and document a statement summarizing those findings. This is called which of the following?

Evaluative statement

How often would a nurse recommend a client eat or drink a source of vitamin C?

Everyday

A client has been admitted to the hospital for the treatment of diabetic ketoacidosis, a problem that was accompanied by a random blood glucose reading of 575 mg/dLm (31.91 mmol/L), vomiting, and shortness of breath. This client has experienced which phenomena?

Exacerbation

A nurse is discharging a client terminates the nurse-client relationship. Which action should the nurse perform in this phase?

Examine goals of the relationship to determine whether they were achieved

A client is unconscious and unable to provide input into outcome identification. With which group of individuals should the nurse consult for the formulation of goals and measurable outcomes?

Family

An elderly client who is wheelchair bound following a cerebrovascular accident is being assessed by the nurse. The nurse notes the client has seepage of stool from the anus. The nurse knows this is indicative of a) Fecal impaction b) Diarrhea c) Intestinal infection d) Constipation

Fecal impaction

Mrs. James comes to her healthcare provider's office because she is having abdominal pain. She has been seen for this problem before. What type of assessment would the nurse do? A. Initial assessment B. Focused assessment C. emergency assessment D. time-lapsed assessment

Focused assessment

The nurse is teaching a client how to take medications upon discharge. The client is alert and oriented, but unable to articulate teaching back to the nurse. What is the appropriate nursing action?

Give written instructions to the client and caregivers.

After collecting data from a patient with respiratory distress, the nurse prioritizes the patient interventions to provide oxygen to the patient first. This is an example of which of the following models of organizing data? A. Hierarchy of Human Needs B. Functional Health Patterns C. Human response patterns D. Body system model

Hierarchy of Human Needs

A nurse is caring for a patient who suddenly begins to have respiratory difficulty. In what position would the nurse place the patient to facilitate respirations?

High Fowler's

The nurse performs a comprehensive assessment of a newly admitted client. What is the primary purpose of this admission assessment?

Identify baseline data

The nurse is assessing a female client for orthostatic hypotension. As the nurse assists the client to a standing position, the client states, "I'm feeling really dizzy." What should the nurse do next?

Immediately assist the client back to bed.

Identifying the kind and amount of nursing services required is a possible solution for:

Inadequate staffing.

The nurse is preparing to educate a client on how to perform incentive spirometry. Which of the following concepts should you include?

Incentive spirometry provides visual reinforcement of deep breathing.

A nurse is educating a client who has congested lungs how to keep secretions thin, and more easily coughed up and expectorated. What would be one self-care measure to teach?

Increase oral intake of fluids to two to three quarts per day.

The nurse is orienting a new client to the facility. The client is told that her preferences and choices would be sought and honored. This represents which expectation of the health care environment?

Individualization

A 10-year-old girl is excited when she receives the "most improved player" award on her softball team. Although she was not the strongest player on the team, she always tried to perform at her best level and never gave up on practicing her skills. What stage of psychosocial development according to Erik Erikson is this child demonstrating?

Industry versus inferiority

A nurse is caring for a toddler who is having an acute asthmatic attack with copious mucus and difficulty breathing. The child's skin is cyanotic, respirations are labored and rapid, and pulse is rapid. What nursing diagnosis would have priority for care of this child?

Ineffective airway

In planning to meet the nutritional needs of a critically ill client in the intensive care unit, which factor will increase the client's basal metabolic rate?

Infection

A nursing student is manually taking the client's blood pressure. Which step will demonstrate the correct way of inflating the blood pressure cuff?

Inflate the cuff to 30 mm Hg above reading where brachial pulse disappeared.

A nurse manager notes an increase in the frequency of client falls during the last month. To promote a positive working environment, how would the nurse manager most effectively deal with this problem? a) Investigate the circumstances that contributed to client falls b) Reprimand the nursing personnel responsible for the clients when the falls occurred c) Institute a new policy on the prevention of client falls on the unit d) Determine if client falls have increased on other nursing units in the hospital

Investigate the circumstances that contributed to client falls

A nurse is caring for an asthmatic client who requires a low concentration of oxygen. Which delivery device should the nurse use in order to administer oxygen to the client?

Nasal cannula

The nurse is conducting group therapy for geriatric patients. The nurse gives instructions to the patients, "Close your eyes and imagine a peaceful scene, you are very comfortable, nothing is there to disturb you." What feedback given by the patients after the therapy indicates effective therapy? The patients Report relief from pain Are able to remain calm and silent Are able to acknowledge their feelings Are able to empathize with their peers

Report relief from pain

A nurse smells smoke and subsequently discovers a fire in a garbage can in a common area on the hospital unit. What is the nurse's priority action in this situation?

Rescue anyone who is in immediate danger.

A patient has had a head injury affecting the brainstem. What is located in the brainstem that may affect respiratory function?

Respiratory Center

The primary purpose for evaluating data about a client's care according to a functional health approach is to

Revise or modify the nursing care plan

A client admitted for a surgical procedure tells the nurse, "I am very worried because I am allergic to latex. I want to make sure that everyone knows this." In order to assure the safety of the client, what nursing diagnosis would the nurse address? a) Knowledge deficit related to surgical procedure b) Risk for allergy response related to latex allergy c) Risk for injury related to latex allergy d) Anxiety related to surgical procedure

Risk for allergy response related to latex allergy

A client has just been admitted to the clinical unit. The nurse is providing her with the expectations she may have of the health care she will receive. She is told that she will not be harmed by any errors that might be made and she can expect to be safe in the facility. This assurance represents which expectation of the health care environment?

Safety

Which reason best explains why adolescents behave in an unsafe manner despite knowledge of a particular activity's risk?

Social pressure

A client states, "I have trouble sleeping. I only sleep about 2 hours and then I wake up." This is:

Subjective data

Nurse Mayweather is auscultating lung sounds. She notes crackles in the LLL which were not present at the start of the shift. Nurse Mayweather is engaged in which type of nursing intervention?

Surveillance intervention

A nurse is caring for a patient who has COPD, a chronic illness of the lungs. The patient is in remission. Which statement best describes a period of remission in a patient with a chronic illness?

Symptoms are not experienced

Which nursing actions reflect the evaluation stage of the nursing process? Select all that apply.

The nurse identifies that a client's pain is not being adequately treated. The nurse documents the client's response to suctioning. The nurse determines the client did not lose the expected 2 lb (0.90 kg).

A nurse follows the universal patient compact principles for partnership when providing care for patients. Which nursing action does not reflect this philosophy?

The nurse makes health care decisions for a patient who is uncooperative.

The nurse is conducting a peer review of a nursing colleague. Which action by the nurse is an example of peer review?

The nurse preceptor provides feedback to the new graduate nurse after 6 weeks of orientation.

Who or what is the primary source of information for a nursing history? A. previous medical records B. other healthcare personnel C. The patient D. Family members

The patient

One of the outcomes that has been identified in the care of a client with a new suprapubic catheter is that he will demonstrate the correct technique for cleaning his insertion site and changing his catheter prior to discharge. When should this outcome be evaluated?

Throughout the client's hospital admission

The health care team has convened to discuss the care of an end-of-life client who is not able to achieve an acceptable level of comfort. The physician asks for the nurse's perspective of the situation. Which standard for establishing and sustaining healthy work environments does this action represent?

True Collaboration

A 14-year-old boy is in the clinic for his well-child exam. When the client asks his mother if she has any questions for the practitioner, she states "He sleeps so much. I am worried about how lazy he is." What does the nurse know to be true about sleep in adolescents?

Trying to balance too many activities can result in sleep deprivation.

What is a dynamic balance among the physical, psychological, social, and spiritual aspects of a person's life?

Wellness

All of the following patients have a body temperature of 38°C (100.4°F). About which patient would a nurse be most concerned? A) an older adult B) a pregnant adolescent C) a junior high football player D) a 2-month-old infant

a 2-month-old infant

During assessment of an older adult's skin integrity, expected findings include which of the following? a. Decreased elasticity b. Oily skin c. Increased facial hair in men d. Faster nail growth

a. Decreased elasticity Loss of skin elasticity is a common finding in the older adult. Other common findings include pigmentation changes, glandular atrophy (oil, moisture, sweat glands), thinning hair (facial hair: decreased in men, increased in women), slower nail growth, and atrophy of epidermal arterioles.

Of all factors, what is the most important risk factor in pulmonary disease?

active and passive cigarette smoke

A nurse is admitting a 6-year-old child status post tonsillectomy to the surgical unit. The nurse obtains his weight and places EKG and a pulse oximeter on the client's left finger. His heart rate reads 100 bpm and the pulse oximeter reads 99%. These readings best indicate:

adequate tissue perfusion

What is the definition of wellness?

an active state of being healthy

The nurse is assessing a newborn in the nursery. The nurse notes the infant has episodes in which breathing ceased for 20 seconds on 2 occasions. The nurse correctly recognizes this condition as:

apnea

The nurse assesses that a client is shivering. Which intervention is most appropriate to prevent further stress on the body?

applying a blanket

What does pulse oximetry measure?

arterial oxygen saturation

Which of the following nursing interventions is an example of tertiary preventative care? a) teaching stress reduction classes at a wellness center b) administration of immunizations to a 6-month-old child c) blood pressure screenings at a senior center d) assisting with speech therapy for a patient with a traumatic brain injury

assisting with speech therapy for a patient with a traumatic brain injury

An elderly patient has been recently diagnosed with vascular dementia. Because he lives alone and has poorly controlled hypertension, he has begun to receive home healthcare. This new aspect of his care is characteristic of which stage of illness? a) assuming a dependent role b) assuming the sick role c) experiencing the symptoms d) achieving recovery and rehabilitation

assuming a dependent role

An 17-year-old is brought to the emergency department with a head injury. The nurse knows that adolescents are vulnerable to injuries related to:

automobile accidents

A nurse collects objective data on a client during a health assessment that includes the client's:

blood pressure

A nurse is volunteering at a day camp. A child is stung by a bee and develops wheezing in the upper airways. The child is experiencing:

bronchospasm.

A new mother is having difficulty breastfeeding her newborn infant. A goal was established stating the baby would be nursing every 2 to 3 hours by age 1 week. The mother presents to the follow-up center at 1 week and reports the she discontinued breastfeeding. The nurse evaluates the original goal as:

completely unmet.

The terms "criteria" and "standards" are often used interchangeably, but they actually have distinct definitions. "Measurable qualities, attributes, or characteristics that identify knowledge or health status" are known as:

criteria.

The nurse understands that accurate blood pressure taking is dependent on several factors. Which example will most likely render an accurate blood pressure reading?

elevating the client's arm at heart level

What type of patient record data would the nurse find in the medical history and progress notes? A. findings of the physician's assessments and treatment B. results of laboratory and diagnostic studies C. nursing documentation and plan of care D. information from other members of the healthcare team

findings of the physician's assessments and treatment

The nurse is conducting a respiratory assessment of a client age 71 years who has been recently admitted to the hospital unit. Which assessment finding should the nurse interpret as abnormal?

fine crackles to the bases of the lungs bilaterally

The nurse is preparing to provide hygiene care to a client with hypoxia. Into what position will the nurse place the client?

high fowlers

The nurse is preparing to assess a client's abdomen. What are the steps of the assessment in the correct order.

inspection auscultation percussion palpation

Which action by the nurse will facilitate the nurse-client relationship during the orientation phase?

introducing oneself to the client by name

A client has come into the clinic for a postoperative visit. The client states that the postoperative pain continues to be 6 on a 0 to 10 rating scale. The nurse evaluates the client and the current plan of care. Based on the information provided by the client, the nurse should:

modify the plan of care.

During a home visit, the nurse learns that the client ensures a daily bowel movement with the help of laxatives. The client feels that deviation from a bowel movement every day is unhealthy. Which nursing diagnosis would the nurse most likely identify? a) Constipation b) Perceived constipation c) Bowel incontinence d) Risk of constipation

perceived constipation

After completing assessment, a nurse uses the data collected to identify appropriate diagnoses for a patient. What are the nursing diagnoses used for?

selecting nursing interventions to meet expected outcomes

What equipment is needed to take an apical pulse? A) sphygmomanometer B) electronic thermometer C) stethoscope D) no specific equipment

stethoscope

A student nurse assesses a blood pressure on an adult and finds it to be 140/86. What term is used for the top number (140)? A) systolic pressure B) diastolic pressure C) pulse pressure D) hypotension

systolic pressure

Various sounds are heard when assessing a blood pressure. What does the first sound heard through the stethoscope represent? A) systolic pressure B) diastolic pressure C) auscultatory gap D) pulse pressure

systolic pressure

A nurse is assessing a patient who has a fever, has an infection of a flank incision, and is in severe pain. What type of pulse rate would be likely? A) bradycardia B) tachycardia C) dysrhythmia D) bigeminal

tachycardia

What is the goal of the nurse in a helping relationship with a client?

to assist the client to identify and achieve goals

A nurse has an order to take the core temperature of a patient. At which of the following sites would a core body temperature be measured? A) tympanic B) oral C) axillary D) skin surface

tympanic

A nurse provides interventions for clients in a long-term care facility to help them meet their intellectual needs. Which nursing actions promote these needs? Select all that apply. a) A nurse manager shuts down a cafeteria to investigate cases of food poisoning. b) A nurse shows residents a video discussing modified activities for older adults. c) A nurse explains to an obese client the benefits of following a healthy diet. d) A nurse provides education about foot care to a client with diabetes. e) A nurse sets up a pet therapy program for the residents. f) A nurse refers a client experiencing dysfunctional grief to a grief counselor.

• A nurse shows residents a video discussing modified activities for older adults. • A nurse explains to an obese client the benefits of following a healthy diet. • A nurse provides education about foot care to a client with diabetes.

Which of the following are examples of nonprofessional involvement? (Select all that apply.)

• Discussing your recent breakup with your boyfriend with a patient who is also going through a difficult breakup. • Asking a patient if they would like to go out for dinner after they are discharged.

A nurse is using the SMART acronym to plan outcomes for patients in a long-term care facility. Which criteria describe the use of this acronym? (Select all that apply.)

• S = goals should be specific • M = goals should be measurable • R = goals should be realistic • T = goals should be temporary

B

10. A resident of a long-term care facility refuses to eat until she has had her hair combed and her make-up applied. In this case, what patient need should have priority? A) the need to have nutrition B) the need to feel good about oneself C) the need to live in a safe environment D) the need for love from others

The nurse is asking admission interview questions and the client has explained the reason for seeking care. What is the most appropriate way to document the response?

Client states, "I feel winded all of the time and yesterday I started spitting up a lot of phlegm."

The nurse is seeing 6-year-old Patrick in the clinic. As the nurse is measuring height and weight for Patrick, his mother comments on how much he has grown since his last checkup. The nurse knows that this increase in size is which of the following?

Growth

When communicating with patients nurses need to be very careful in their approach. This is particularly true when communicating using

Medical terminology

A nutritionist helps to plan a diet for a client with diabetes. Which of the following foods is a carbohydrate that should be included to help improve glucose tolerance?

Oatmeal

Which physiologic responses are associated with severe long-term stress? Select all that apply. Obesity Hypotension Atherosclerosis Protein breakdown Stimulation of the immune system

Obesity Atherosclerosis Protein breakdown

At the end of the shift, the nurse documents that the patient has voided 475 mL during the shift via an indwelling urinary catheter. What type of data has the nurse documented?

Objective

A client is being treated for chronic obstructive pulmonary disease. The nurse auscultates the client's lungs following a period of coughing. The findings of this assessment are an example of:

Objective data

The nurse is caring for Mr. M., a 48-year-old man with congestive heart failure. The nurse manager informs the nurse that Mr. M. was enrolled in a clinical trial to assess whether a 10-minute walk, 3 times per day, leads to expedited discharge. Which type of evaluation best describes what the researchers are examining?

Outcome evaluation

A nurse is caring for a critically ill client who is receiving oxygen through a non-rebreather mask. The nurse should remember that which of the following situations could lead to oxygen toxicity in the client?

Oxygen concentration of more than 50% given for longer than 48 hours

A nurse assesses the abdomen of a client before and after administering a small-volume cleansing enema. What condition would be an expected finding? a) Abdominal tenderness b) Areas of distention c) Increased bowel sounds d) Muscular resistance

Increased bowel sounds

A nurse has documented that a client has anorexia. What does this term mean?

Lack of appetite

The type of stool that will be expelled into the ostomy bag by a client who has undergone surgery for an ileostomy will be a) Soft semi-formed b) Bloody c) Liquid consistency d) Mucus filled

Liquid consistency

Over a 5-year period, an adult experienced liver failure. A liver transplant was performed. During the immediate postoperative period, which intervention could the nurse recommend to enhance healing and reduce the harmful effects of stress? Aerobic exercise Listening to music Attending a support group Occasional chocolate treats

Listening to music

What is the primary role of the nurse in the care of clients that experience domestic violence?

Providing prompt recognition of the potential or actual threat to safety

A nurse is evaluating the outcome of the plan of care after teaching a client how to prepare and administer an insulin pen. Which of the following types of outcome is the nurse addressing?

Psychomotor

Before discharge the client will demonstrate aseptic dressing changes. This is an example of which type of evaluative statement?

Psychomotor

The client demonstrates stair climbing using a quad cane. What type of outcome is this an example of?

Psychomotor outcome

A nurse responds to the call bell and finds another nurse evacuating the client from the room, which has caught fire. Which action should the nurse take?

Pull the fire alarm lever.

A patient returns to the telemetry unit after an operative procedure. Which of the following diagnostic tests will the nurse perform to monitor the effectiveness of the oxygen therapy ordered for the patient?

Pulse Oximetry

The nurse is discussing diabetes mellitus with the family members of a client recently diagnosed. In order to promote the health of the family members, what would be the most important information for the nurse to include? a) Medications used to treat diabetes mellitus b) The cellular metabolism of glucose c) Risk factors and prevention of diabetes mellitus d) The severity of the client's disease

Risk factors and prevention of diabetes mellitus

Why is communication important to the "assessment" step of the nursing process?

The major focus of assessing is to gather information.

As the nurse is preparing an immunization for a male client 2 years of age, his mother discovers that he has soiled his underpants. She scolds him and calls him a "bad boy" because he "dirtied" his underwear. The nurse gives the immunization and provides some education to the mother about appropriate expectations for this age group. The nurse knows that according to Erikson's theory for this client's age group, he is at risk to develop which of the following?

Shame and doubt

For a client with self-care deficit, the long-term goal is that the client will be able to dress himself by the end of the 6-week therapy. For best results, when should the nurse evaluate the client's progress toward this goal?

as soon as possible

Which symptom is an expected cognitive change in the older adult patient? a. Disorientation b. Slower reaction time c. Poor judgment d. Loss of language skills

b. Slower reaction time Slower reaction time is a common change in the older adult owing to degeneration of nerve cells, decreased neurotransmitters, and decreased rate of conduction of impulses. Symptoms of cognitive impairment, such as disorientation, loss of language skills, loss of the ability to calculate, and poor judgment are not normal aging changes and require further investigation of underlying causes.

A patient asks the nurse what the term polypharmacy means. The nurse defines this term as a. Multiple side effects experienced when taking a medication. b. The concurrent use of many medications. c. The many adverse drug effects reported to the pharmacy. d. The risks of medication effects due to aging.

b. The concurrent use of many medications. Polypharmacy refers to the concurrent use of many medications. It does not have anything to do with side effects, adverse drug effects, or risks of medication use due to aging.

`A nursing student caring for an unconscious client knows that communication is important even if the client does not respond. Which nonverbal action by the nursing student would communicate caring?

holding the client's hand while talking

A male client has always prided himself in maintaining good health and is consequently shocked at his recent diagnosis of diabetes. The nurse has asked the client, "How do you think your diabetes is going to affect your lifestyle?" The nurse has utilized which of the following interviewing techniques?

open-ended question

What site for taking body temperature with a glass thermometer is contraindicated in patients who are unconscious? A) rectal B) tympanic C) oral D) axillary

oral

A client presents to the Emergency Department with a temperature of 100.6F (38.1°C) and BP of 108/60 mm Hg. What intervention does the nurse anticipate providing?

oral fluids

Which of the following is an average normal temperature in Centigrade for a healthy adult? A) oral: 37.0°C B) rectal: 36.5°C C) axillary: 37.5°C D) tympanic: 34.4°C

oral: 37.0°C

The nurse is explaining primary and secondary care to the client. Which nursing activity reflects secondary care as provided by the nurse?

ordering a mammography

A nurse is evaluating a client's orientation after he was brought into the ER following a car accident. What is indicated by "Oriented x3"?

oriented to person, place, and time

A nurse is caring for a patient who is ambulating for the first time after surgery. Upon standing, the patient complains of dizziness and faintness. The patient's blood pressure is 90/50. What is the name for this condition? A) orthostatic hypotension B) orthostatic hypertension C) ambulatory bradycardia D) ambulatory tachycardia

orthostatic hypotension

The focus of a hospital's current quality assurance program is a comparison between the health status of clients upon admission and at the time of discharge. This form of quality assurance is characteristic of:

outcome evaluation.

The arterial blood gases for a patient in shock demonstrate increased carbon dioxide and decreased oxygen. What type of respirations would the nurse expect to assess based on these findings? A) absent and infrequent B) shallow and slow C) rapid and deep D) noisy and difficult

rapid and deep

A nurse is assessing the blood pressure on an obese woman. What error might occur if the cuff used is too narrow? A) reading is erroneously high B) reading is erroneously low C) pressure on the cuff with be painful D) it will be difficult to pump up the bladder

reading is erroneously high

An adult patient is assessed as having an apical pulse of 140. How would the nurse document this finding? A) bradycardia B) tachycardia C) dysrhythmia D) normal pulse

tachycardia

The nurse is preparing a plan of care for a client with nutritional deficits. Which is the priority intervention for this client?

teaching about intake of food and vitamins

Chronic illness may be characterized by periods of remission. Remission is best defined as: a) the response of a person to a disease. b) the presence of a disease with the absence of symptoms. c) a pathologic changes in the structure of function of the body or mind. d) the reappearance of symptoms of a disease.

the presence of a disease with the absence of symptoms.

On admission, a physician diagnoses a patient with rheumatoid arthritis. The nurse uses assessments to make the nursing diagnosis of chronic pain. What is the nurse diagnosing? A. the pathology of the illness B. the response of the patient of the illness C. information from a nursing textbook D. knowledge from more experienced nurses B

the response of the patient of the illness

A nurse is teaching a home care patient and his family about using prescribed oxygen. What is a critical factor that must be included in teaching?

the safety measures necessary to prevent a fire

One of the outcomes that has been identified in the care of a client with a new suprapubic catheter is that he will demonstrate the correct technique for cleaning his insertion site and changing his catheter prior to discharge. When should this outcome be evaluated?

throughout the client's hospital admission

The nurse completes a health history and physical assessment on a patient who has been admitted to the hospital for surgery. What is the purpose of this initial assessment? A. to gather data about specific and current health problem B. to identify life threatening problems that require immediate attention C. to compare and contrast current health status to baseline data D. to establish a database to identify problems and strengths

to establish a database to identify problems and strengths

A nurse is assisting with lunch at a nursing home. Suddenly, one of the residents begins to choke and is unable to breathe. The nurse assess the resident's ability to breathe and then begins CPR. Why did the nurse assess respiratory status? A. to identify a life-threatening problem B. to establish a database for medical care C. to practice respiratory assessment skill D. to facilitate the resident's ability to breathe

to identify a life-threatening problem

What is the primary purpose of validation as part of assessment? A. to identify data to be validated B. to establish an effective nurse-patient communication C. To maintain effective relationships with coworkers D. to plan appropriate nursing care

to plan appropriate nursing care

A nurse is assessing a new client's level of activity and exercise. What should be addressed with every client?

whether they have a program of regular physical activity

A mother is bringing her infant into the the clinic for a well baby check. The infant's weight gain is on target for age. A correctly written evaluative statement is which of the following?

"8FEB2016. Goal met. The infant's weight gain is appropriate for age."

An anxious son asks you how he can keep his elderly father safe in his home. He tells you his father lives alone, has chronic illnesses, and also has sensory-perceptual alterations. What is the best statement by the nurse?

"A good idea is to get a weekly medicine tray at the pharmacy. This will keep him from missing medication doses or taking the wrong pill by accident."

Which safety tip could the nurse give to parents to help decrease the risk of the leading cause of injury or death in children 1 to 4 years of age?

"Always provide close supervision for young children when they are in or around pools and bathtubs."

Which of the following questions or statements would be an appropriate termination of the health history interview? A. "Well, i can't think of anything else to ask you right now." B. "Can you think of anything else you would like to tell me?" C. "I wish you could have remembered more about your illness." D. "Perhaps we can talk again sometime. Goodbye."

"Can you think of anything else you would like to tell me?"

A father of a preschool-aged child tells the nurse that his child "has had a constant cold since going to daycare." How would the nurse respond?

"Children in daycare have more exposure to colds."

When recording or documenting outcome attainment in the chart, nurses are to be very clear with the descriptions used. Which term is appropriate?

"Demonstrated steps"

A nurse is conducting a health history interview for a woman at an assisted-living facility. The woman says, "I have been so constipated lately." How should the nurse respond? A. "Do you have a family history of chest problems." B. " Why don't you use a laxative every night?" C. "Do you take anything to help your constipation?" D. " Everyone who ages has bowel problems."

"Do you take anything to help your constipation?"

A client has been recently diagnosed with type 1 diabetes mellitus. He is seen in the emergency room every day with high blood sugar. The client apologizes to the nurse for bothering them every day, but he cannot give himself insulin injections. What should the nurse's response be?

"Has someone taught you how to take them?"

A large health care organization has committed to promoting a just culture when adverse events and near misses take place. Which question will guide the organization's response when a nurse commits an error?

"How did the nurse's actions contribute to this error?"

Which assessment question focuses on determining the resilience of a preteen child? Select all that apply. "Do you like playing video games?" "How do you and your friends have fun?" "How are you adjusting to your new school?" "What would you do if you forgot your homework?" "What job would you like when you grow up?"

"How do you and your friends have fun?" "How are you adjusting to your new school?" "What would you do if you forgot your homework?" "What job would you like when you grow up?"

A nurse caring for a client who was recently diagnosed with metastatic lung cancer finds the client crying in the room. Which statement made by the nurse best demonstrates the use of empathy?

"I see you are upset. Would you like to talk?"

The expected outcome for a client with a new diagnosis of diabetes mellitus (DM) is: client will describe appropriate actions when implementing the prescribed medication routine. Which statement by the client indicates the outcome expectation has been met?

"I will test my glucose level before meals and use sliding scale insulin."

An 8-year-old boy fell off his bicycle. He was not wearing a helmet and has sustained a concussion. What information should the nurse teach the parents about concussions?

"It is important to monitor frequently for headache, vomiting, visual disturbances, and changes in alertness."

A nurse is collecting information from Mr. Koeppe, a patient with dementia. The patient's daughter, Sarah, accompanies the patient. Which of the following statements by the nurse would recognize the patient's value as an individual? A. "Sarah, can you tell me how long your father has been this way" B. "Sarah, i have to go and read your father's old charts before we talk" C. "Mr.Koeppe, tell me what you do to take care of yourself." D. "Mr. Koeppe, i know you can't answer my questions, but its okay."

"Mr.Koeppe, tell me what you do to take care of yourself."

A nurse is explaining the purpose of nursing diagnoses to a client. What would be the most appropriate statement for the nurse to make? a) "Nursing diagnoses are used to bill insurance for nursing care." b) "Nursing diagnoses are necessary to schedule the amount of nursing care required by the client." c) "Nursing diagnoses are used to guide the nurse in selecting appropriate nursing interventions." d) "Nursing diagnoses are necessary to validate the medical diagnosis."

"Nursing diagnoses are used to guide the nurse in selecting appropriate nursing interventions."

The nurse teaches a patient with anxiety about stress and triggers. Which statement made by the patient indicates the teaching was effective? "Stress is a negative response." "Eustress is resistance to stress." "People can have good and bad stress." "People with stress are always in distress."

"People can have good and bad stress."

A nursing student asks the clinical instructor to explain the difference between quality improvement and quality assurance. Which response by the clinical instructor is appropriate?

"Quality improvement focuses on processes, data, and statistical thinking."

A client is discussing weight loss with a nurse. The patient says, "I will not eat for two weeks, then I will lose at least 10 pounds." What should the nurse tell the client?

"That will decrease your metabolic rate and make weight loss more difficult."

The nurse is assessing an adolescent with an annual physical. The mother reports that she has noticed a change in the child's behavior lately including mood swings, withdrawal from the family, and failing school grades. The mother does not know what to do and asks the nurse for guidance. What is the most appropriate guidance from the nurse?

"These could be signs of substance abuse. Open communication and a referral to a counselor that specializes in substance abuse would be beneficial."

A nurse in the emergency department is completing an emergency assessment for a teenager just admitted from a car crash. Which of the following is objective data? A. "My leg hurts so bad. I can't stand it." B. "Appears anxious and frightened." C. "I am so sick;I am about to throw up" D. "Unable to palpate femoral pulse in left leg."

"Unable to palpate femoral pulse in left leg."

Which statement indicates that a family understands the teaching that has been provided by the nurse related to car seat safety for their 3-year-old child?

"We place our child in a front-facing car seat in the back seat of the car."

The nurse is interviewing a client to obtain the health history. Which question would the nurse ask first?

"What brings you here today?"

A nurse is feeding a client. Which of the following statements would help a person maintain dignity while being fed?

"What part of your dinner would you like to eat first?"

An 82-year-old male client admitted 4 days ago is being treated for chronic obstructive pulmonary disease (COPD) and is being cared for by the nursing assistive personnel (NAP). The NAP comes to the nurse and tells her that her client seems more confused today than yesterday. To assess the client's level of orientation, which of the following questions would the nurse ask?

"What's the name of this hospital?" (ask open-ended questions)

A nurse is examining a 3-year-old child with conjunctivitis. During the examination, the child starts crying and refuses to sit still. Which of the following statements is appropriate for the nurse tell the child?

"Would you like to see my flashlight?"

A nurse is having problems communicating with a client. Which statement by the nurse would open up the most dialogue with the client?

"You are back from therapy; tell me about it."

During a home visit, the nurse finds that an adolescent is upset after failing a midterm exam at school. What positive statement does the nurse use to comfort the adolescent? "Do you feel like you should have studied more?" "You should not be upset for such a small reason." "This probably just isn't your best subject, and you should focus on your stronger subjects." "You can learn from this experience and refocus your attention to improve your future performance."

"You can learn from this experience and refocus your attention to improve your future performance."

The wife of a client who is terminally ill expresses to the nurse that she is unable to see her husband die and she may not come to the health care facility anymore. What should the nurse's response to her be?

"You have been coming here every day; are you taking some time for yourself?"

A client who uses portable home oxygen states, "I still like to smoke cigarettes every now and then." What is the appropriate nursing response?

"You should never smoke when oxygen is in use."

Which of the following nutritional guidelines should a nurse provide to a client who is entering the second trimester of her pregnancy?

"You'll need to eat more calories and to make sure you eat a balanced diet high in nutrients."

A nurse teaches a client deep-breathing exercises to help control his anxiety. This is considered what type of stress management technique? - A) Meditation - B) Relaxation - C) Anticipatory guidance - D) Guided imagery

- Relaxation

A friend has lost her job and is becoming increasingly anxious to the point of crisis. What type of crisis is she experiencing? - A) Adventitious - B) Maturational - C) Situational - D) Emotional

- Situational

What questions would be helpful in eliciting data about the effects of stress during a health history? - A) "Why are you having so much difficulty breathing at night?" - B) "Why do you think smoking and drinking will calm you?" - C) "Do you often drink too much and have hangovers?" - D) "How does your body feel when you are upset?"

- "How does your body feel when you are upset?"

A nurse is educating a client about the benefits of exercise in reducing stress. How often would the nurse recommend the client exercise? - A) Two hours every day - B) One hour once a week - C) 30 to 45 minutes, most days of the week - D) 60 to 75 minutes, four to five times a week

- 30 to 45 minutes, most days of the week

Which of the following responses to stressors results from the activation of the local adaptation syndrome? - A) A girl quickly withdraws her hand from a stream of hot tap water. - B) A woman's impending job interview has prompted the activation of her fight-or-flight response. - C) A man is experiencing moderate anxiety before meeting with an important client. - D) A man has a sudden urge for a bowel movement before undergoing thoracentesis.

- A girl quickly withdraws her hand from a stream of hot tap water

Which of the following is an example of developmental stress? - A) A newborn who needs to be fed by bottle - B) A school-age child learning to read - C) A teenager learning to drive a car - D) A middle adult accepting signs of aging

- A middle adult accepting signs of aging

Which of the following best describes stress? - A) A response to changes in the normal balanced state - B) A perception that something is threatening - C) A response to internal environment for homeostasis - D) A localized response of a tissue or organ to a stressor

- A response to changes in the normal balanced state

What is the term for the change that takes place in response to a stressor? - A) Rehabilitation - B) Adaptation - C) Positive movement - D) Negative movement

- Adaptation

Which diagnoses would be written for stress as the cause of the problem? - A) Anxiety related to conflicts about values and goals in life - B) Caregiver Role Strain related to long-term stress of care for a parent with Alzheimer's disease - C) Spiritual Distress related to inability to accept diagnosis of terminal illness - D) Hopelessness related to presence of disabling physical injuries

- Anxiety related to conflicts about values and goals in life

Which of the following statements, made by a senior citizen who has taken a class on stress reduction, would indicate to the nurse the need for further instruction? - A) Adults draw on coping skills learned throughout life. - B) Family members can be supportive during stress. - C) Stress may be positive or negative. - D)As one grows older, their stress decreases.

- As one grows older, their stress decreases.

the primary controllers of homeostatic mechanisms? - A) Autonomic nervous system - B) Endocrine system - C) Respiratory system - D)Cardiovascular system - E) Gastrointestinal system

- Autonomic nervous system - Endocrine system

An individual steps into a tub of very hot water and immediately jumps out again. What mechanism caused this response? - A) Inflammatory response - B) Reflex pain response - C) General adaptation syndrome - D) Fight-or-flight response

- Reflex pain response

Which of the following illnesses has been associated with long-term stress? - A) Bacterial infections - B) Cardiovascular disease - C) Renal disease - D) Fractures

- Cardiovascular disease

The nurse walks into the client's room and finds her sobbing uncontrollably. When the nurse asks what the problem is, the client responds I am so scared. I have never known anyone who goes into a hospital and comes out alive. On this client's care plan the nurse notes a nursing diagnosis of Ineffective coping related to stress. What is the best outcome you can expect for this client? - A) Client will adapt relaxation techniques to reduce stress. - B) Client will be stress free. - C) Client will avoid stressful situations. - D) Client will start anti-anxiety agent.

- Client will adapt relaxation techniques to reduce stress.

A client responds to bad news regarding test results by crying uncontrollably. What is the term for this response to a stressor? - A) Adaptation - B) Homeostasis - C) Coping mechanism - D) Defense mechanism

- Coping mechanism

A man has noticed bright red blood in his bowel movements for over a month. He says to himself, "Oh, it's just my hemorrhoids." What defense mechanism is the man using? - A) Rationalization - B) Repression - C) Denial - D) Compensation

- Denial

Various physiologic mechanisms within the body respond to internal changes to maintain relative constancy in the internal environment. The state that results is called what? - A) Nirvana - B) Homeostasis - C) Fight-or-flight response - D) Anxiety

- Homeostasis

If a nurse assessed the vital signs of a person who was in the initial alarm reaction stage (shock phase) of the GAS, what would be the expected findings? - A) Slow, deep breathing - B) Fatigue and lethargy - C) Hypotension - D) Hypertension

- Hypertension

A client with an inflamed appendix is feeling pain in the stomach area. What is the term for this body response to stress? - A) Local adaptation syndrome - B) General adaptation syndrome - C) Physiological homeostasis - D) Fight-or-flight response

- Local adaptation syndrome

A student is preparing for her first client care assignment. She wakes up at 4 AM with a pounding pulse and diarrhea. What type of adaptive response to stress is she experiencing? - A) General adaptation syndrome - B) Mind-body interaction - C) Local adaptation syndrome - D) Coping or defense mechanism

- Mind-body interaction

Which area of nursing experiences the highest levels of stress in the work setting? - A) Obstetric nurses - B) Pediatric nurses - C) New graduates - D) Aging nurses

- New graduates

A nurse is preparing to educate a client about care at home. On entering the room, she finds the client pacing around the room, hyperventilating, and complaining of nausea. Based on these manifestations of severe anxiety, what would the nurse do? - A)Provide both verbal and written information to the client. - B) Ignore the client and teach the family the information. - C) Modify the education plan to the client's anxiety level. - D) Postpone implementation of the education plan.

- Postpone implementation of the education plan

The wife of a client on hospice at home is diagnosed with "caregiver burden." Which of the following best describes this syndrome? - A) Prolonged stress from caring for a family member at home - B)Inability to provide competent care for a family member - C)Insufficient funds to pay for medical care of a family member - D) Effect of the illness causing stress in siblings

- Prolonged stress from caring for a family member at home

A client who is being seen at a physician's office states that he has "bad headaches all the time." Diagnostic tests are normal. What is this type of response to stress called? - A) Psychosomatic disorder - B) Acute illness - C) Chronic illness - D) Pretend disorder

- Psychosomatic disorder

What phase of the general adaptation syndrome is a client in when he uses all of his adaptive mechanisms for dealing with stress, leaving no defense against the distress? - A) Alarm reaction stage - B) Fight-or-flight stage - C) Stage of resistance - D) Stage of exhaustion

- Stage of exhaustion

The client is a single parent being seen at an employee wellness clinic. The client tells the nurse he has three school-age children who participate in various activities. He states, "Every night, I am taking them to soccer practice, football practice, Girl Scouts. I have to fix dinner and then go over homework. I have no time for myself. I am so tired." The client exhibits tremors of his hands. Blood pressure is 140/88 mm Hg, heart rate is 98 beats/minute, and respirations are 30 breaths/minute. Data best supports which of the following nursing diagnoses? - A) Stress Overload related to single parenting - B) Defensive Coping related to inability to appraise stressors - C) Hopelessness related to multiple responsibilities - D) Disturbed Sleep Pattern related to anxiety

- Stress Overload related to single parenting

When discussing his problem, a client tells the nurse that he is always doing small, petty jobs for everyone and he is not happy about it. Because of this, he is feeling stressed and has been getting into fights with his wife. What should the nurse suggest to help the client overcome this problem? - A) Change jobs. - B) Avoid people who dump tasks on him. - C) Take control of the situation. - D) Avoid doing petty jobs.

- Take control of the situation

A client has been brought to the health care facility with accident-related injuries. During the initial interview, the client becomes agitated, upset and is unable to answer any more of the nurse's questions. What does the nurse conclude about the condition of the client? - A) The client's mind is preparing for a fight-or-flight response as he relates the incident. - B) The client's mind is numb, and he is not able to react to further questions from the nurse. - C) The brain is receiving less oxygen with each passing minute, which does not allow him to speak. - D) The brain is sending chemicals to the bloodstream that make the client afraid of the questions.

- The client's mind is preparing for a fight-or-flight response as he relates the incident.

Cold temperatures and loud noises are stressors to one person but not another. Why does this occur? - A)Although the perception is the same, the response is in dividualized. - B) Both individuals will respond the same, depending on the situation. - C) The perception and effects of stressors are highly individualized. - D) The internal environment of one person is more selective

- The perception and effects of stressor are highly individualized.

A client age 35 years is stressed because he is having problems with his superiors at work. The client says that he had an argument with his superiors about his salary. He has become very anxious and has started consuming large amounts of caffeine very frequently. He also suffers from insomnia and gets angry quickly. Which of the following techniques should the nurse suggest to help the client? - A) Non-therapeutic coping strategy - B) Negative coping strategy - C) Therapeutic coping strategy - D) Sensory manipulation strategy

- Therapeutic coping strategy

Which of the following group of terms best describes anxiety? - A) Cognitive, known threat, depression - B) Cognitive, visible threat, anger - C) Known source, prolonged, solely physical - D) Unknown cause, emotional, apprehensive

- Unknown cause, emotional, apprehensive

The client is a child age 5 years hospitalized for a procedure. The client is bedwetting. The parents report this is a new behavior and their child is toilet trained. The nurse assesses the client is exhibiting the defense mechanism of ... - A) compensation. - B) displacement. - C) reaction formation. - D) regression.

- regression.

The nurse is collecting health data and avoids using closed-ended questions. Which are examples of closed-ended questions? (Select all that apply.)

-"Does it hurt when I touch you here?" -"Is there any chance you might be pregnant?" -"Are you ready to get out of bed?" -"Do you smoke cigarettes?"

The nurse is providing discharge teaching to a client going home with oxygen therapy. Which statements made by the client would indicate to the nurse that the teaching was effective? Select all that apply.

-"I will not allow smoking within 10 feet (3 meters) of my oxygen." -"I will keep the oxygen tank away from direct sunlight or heat."

The nurse has presented an educational in-service about caring for clients who have newly created ostomies. The nurse asks participants, "How will you know when a client begins to accept the altered body image?" Which of the following responses by participants indicates a correct understanding of the material? Select all that apply. a) "The client is willing to look at the stoma." b) "The patient uses spray deodorant several times an hour to mask odor." c) "The client makes neutral or positive statements about the ostomy." d) "The client agrees to take prescribed antidepressants." e) "The client expresses interest in learning self-care."

-"The client is willing to look at the stoma." - "The client makes neutral or positive statements about the ostomy." -"The client expresses interest in learning self-care."

The nurse is caring for a client who underwent abdominal surgery today. Which nursing diagnoses would be appropriate for the nurse to utilize? Select all that apply.

-Acute Pain related to disruption of skin tissues secondary to abdominal surgery -Risk for Infection related to altered tissue integrity -Impaired Mobility related to fear of pain -Risk for Constipation related to immobility

The nurse is preparing to measure a client's rectal temperature. Which supplies and equipment should the nurse have available before beginning the procedure? Select all that apply.

-An electronic thermometer with a rectal probe Disposable probe cover -Water-soluble lubricating gel

A nurse is applying the nursing process and is in the diagnosis phase. With which activities would the nurse be involved? Select all that apply.

-Analyzing data -Identifying patterns -Identifying indicators of potential dysfunction

A nurse is formulating the care plan for a client. Which nursing orders should the nurse include in the nursing plan to help bring about the desired outcome in the client? Select all that apply

-Educating the client about his or her condition -Making referrals to support groups for the client -Making further assessments on the client's condition

A client has been admitted with symptoms of shortness of breath on exertion, edematous lower extremities, extreme fatigue, and hypertension. Which are priority nursing diagnoses? Select all that apply.

-Excess fluid volume -Decreased cardiac output -Activity intolerance

Which of the following factors increase BMR? Select all that apply.

-Growth -Infections -Fever -Emotional Tension

A nurse is caring for older adults in a nursing home. Which of the following age-related changes may affect the respiratory functioning of the patients living there? Select all that apply.

-Less air exchange, more secretions in lungs -Greater risk for aspiration due to slower gastric motility -Impaired mobility and inactivity effects the medication

A student nurse is assisting the school nurse with a health fair at a middle school. The student nurse assesses the students' heights and weights as they file through the station, observing that there is a large variation in physical size and emotional maturity. Which of the following factors may affect emotional growth of children? Choose all that apply.

-Loving caregivers -Praise for doing well -Development of trust

A client visits the health care facility for a scheduled physical assessment. What should the nurse do when physically assessing the quality of the client's oxygenation? Select all that apply.

-Monitor the client's respiratory rate. -Check the symmetry of the client's chest. -Observe the breathing pattern and effort.

When planning for discharge planning of a client, which steps should a nurse take? Select all that apply

-Note resolved problems on the care plan -Suggest revisions for unmet goals by the client -Set new goals after achieving previous goals

The nurse is providing prenatal education for a group of pregnant teenagers. The nurse talks about substance abuse during pregnancy and relates which of the following possible newborn consequences? Choose all that apply.

-Premature birth -Congenital anomalies -Low birth weight

The nurse is providing discharge education for a client diagnosed with hypertension. Which teaching points about monitoring blood pressure should the nurse include in the plan? Select all that apply. a. Use the blood pressure devices in public places to measure BP whenever possible. b. Recommend taking the blood pressure every day at the same time. c. Recommend a cuff size appropriate for the client's limb size. d. If using a forearm monitor, tell the client to keep wrist at heart level when using it.

-Recommend taking the blood pressure every day at the same time. -Recommend a cuff size appropriate for the client's limb size. -If using a forearm monitor, tell the client to keep wrist at heart level when using it.

Which client outcome is a cognitive outcome? Select all that apply.

-The client lists the side effects of digoxin (Lanoxin). -The client describes how to perform progressive muscle relaxation. -The client identifies signs and symptoms of hypoglycemia.

A nurse is communicating the plan of care to a client who is cognitively impaired. Which nursing actions facilitate this process? Select all that apply.

-The nurse shows patience with the client and gives the client time to respond. -The nurse maintains eye contact with the client. -The nurse keeps communication simple and concrete.

Which of the following data regarding a patient with a diagnosis of colon cancer are subjective? Select all that apply

-The patient's chemotherapy causes him nausea and loss of appetite -The patient has been experiencing fatigue in recent weeks

Which of the following statements accurately describe a step for inserting an oropharyngeal airway? Select all that apply.

-Use an airway that is the correct size (size 90 mm is appropriate for the average adult). -Airway should reach from opening of mouth to the back angle of the jaw. - Open patient's mouth by using your thumb and index finger to gently pry teeth apart. -Insert the airway with the curved tip pointing up toward the roof of the mouth.

D

1. What is the primary purpose of the outcome identification and planning step of the nursing process? A) to collect and analyze data to establish a database B) to interpret and analyze data to identify health problems C) to write appropriate patient-centered nursing diagnoses D) to design a plan of care for and with the patient

A,C,D,F

11. In which of the following patients has the order of priorities for nursing diagnoses changed? Select all that apply. A) a patient in a long-term care facility who had a stroke B) a patient who is recovering from a broken leg C) a patient who insists on using the bathroom instead of a bedpan D) a patient who appears confused after taking pain medication E) a pregnant patient whose contractions are progressing as anticipated F) a patient who has wounds that require stitches as well as a concussion

C

12. From what part of the nursing diagnoses are outcomes derived during outcome identification and planning? A) the defining characteristics B) the related factors C) the problem statement D) the database

C

13. A nurse writes down the following outcome for a depressed patient: By 6/9/12, the patient will state three positive benefits of receiving counseling. This is an example of which of the following types of outcomes? A) psychomotor B) cognitive C) affective D) realistic

B

15. A nurse is developing outcomes for a specific problem statement. What is one of the most important considerations the nurse should have? A) that the written outcomes are designed to meet nursing goals B) to encourage the patient and family to be involved C) to discourage additions by other healthcare providers D) why the nurse believes the outcome is important

C

16. Which of the following outcomes is correctly written? A) Abdominal incision will show no signs of infection. B) On discharge, patient will be free of infection. C) On discharge, patient will be able to list five symptoms of infection. D) During home care, nurse will not observe symptoms of infection.

B,D,F

17. Which of the following are verbs that are helpful in writing measurable outcomes? Select all that apply. A) know B) define C) hear D) verbalize E) feel F) list

C

18. Which of the following illustrates a common error when writing patient outcomes? A) Patient will drink 100 mL of fluid every 2 hours from 6 a.m. to 9 p.m. B) Patient will demonstrate correct sequence of exercises by next office visit. C) Patient will be less anxious and fearful before and after surgery. D) On discharge, patient will list five symptoms of infection to report.

The community health nurse is talking with four clients. Who does the nurse identify that would most benefit from teaching about alcohol and drug abuse?

19-year-old male college student majoring in physics

B

19. Which of the following groups of terms best describes a nurse-initiated intervention? A) dependent, physician-ordered, recovery B) autonomous, clinical judgment, patient outcomes C) medical diagnosis, medication administration D) other healthcare providers, skill acquisition

B

2. Critical thinking is an essential component in all phases of the nursing process. What question might be used to facilitate critical thinking during outcome identification and planning? A) How do I best cluster these data and cues to identify problems? B) What problems require my immediate attention or that of the team? C) What major defining characteristics are present for a nursing diagnosis? D) How do I document care accurately and legally?

C

20. What part of the nursing diagnosis statement suggests the nursing interventions to be included in the plan of care? A) problem statement B) defining characteristics C) etiology of the problem D) outcomes criteria

B

21. What is true of nursing responsibilities with regard to a physician-initiated intervention (physician's order)? A) Nurses do not carry out physician-initiated interventions. B) Nurses do carry out interventions in response to a physician's order. C) Nurses are responsible for reminding physicians to implement orders. D) Nurses are not legally responsible for these interventions.

A

22. A nurse is using a structured care methodology that follows a set of steps based on a clinician's decision process to help standardize nursing care plans. What is the term for this element of a structured care methodology? A) algorithm B) national guidelines C) standard of care D) clinical practice guideline

C

23. What name is given to tools that are used to communicate a standardized interdisciplinary plan of care for patients within a case management healthcare delivery system? A) Kardex care plans B) computerized plans of care C) clinical pathways D) student care plans

B

24. A nurse has developed a plan of care with nursing interventions designed to meet specific patient outcomes. The outcomes are not met by the time specified in the plan. What should the nurse do now in terms of evaluation? A) Continue to follow the written plan of care. B) Make recommendations for revising the plan of care. C) Ask another healthcare professional to design a plan of care. D) State goal will be met at a later date.

C

25. A nurse records patient data on a folded card and places it in a central file, where it is easily accessible to staff. Which system of care is this nurse using? A) critical pathways B) case management C) Kardex care plan D) concept map care plan

D

26. Which of the following types of care plans is most likely to enable the nurse to take a holistic view of the patient's situation? A) Kardex B) case management C) critical pathways D) concept map care plan

A

28. What common problem is related to outcome identification and planning? A) failing to involve the patient in the planning process B) collecting sufficient data to establish a database C) stating specific and measurable outcomes based on nursing diagnoses D) writing nursing orders that are clear and resolve the problem

A,B,E

29. Which of the following statements accurately describe the impact on nursing of using NIC/NOC standardized languages? Select all that apply. A) They demonstrate the impact that nurses have on the system of healthcare delivery. B) They standardize and define the knowledge base for nursing curricula and practice. C) They limit the number of appropriate nursing intervention to be selected. D) They hinder the teaching of clinical decision making to novice nurses. E) They enable researchers to examine the effectiveness and cost of nursing care. F) They slow the development and use of nursing information systems.

A nurse teaches adults preventive measures to avoid problems of middle adult years. Which of the following are the major health problems during the middle adult years? A) cardiovascular disease, cancer B) upper respiratory infections, fractures C) communicable diseases, dementia D) sexually transmitted diseases, drug abuse

A

B,C,D

3. Nurses identifying outcomes and related nursing interventions must refer to the standards and agency policies for setting priorities, identifying and recording expected patient outcomes, selecting evidence-based nursing interventions, and recording the plan of care. Which of the following are recognized standards? Select all that apply. A) professional physicians' organizations B) state Nurse Practice Acts C) The Joint Commission D) the Agency for Health Care Research and Quality E) the Patient Health Partnership F) the Patient Bill of Rights

A nurse is delivering 3 L/min oxygen to a client via nasal cannula. What percentage of delivered oxygen is the client receiving?

32%

B

4. A nurse admits a patient to the hospital's short-stay unit and completes a health history and physical assessment. Using these data, the nurse develops a(n) ___________plan of care, based on _____________ planning? A) intermittent, focused B) comprehensive, initial C) single-use, ongoing D) standard, emergency

D

5. Although each care plan is individualized, there are certain risks and health problems that, for example, patients undergoing similar medical or surgical treatment have in common. What name is given to this type of care plan? A) initial B) ongoing C) discharge D) standardized

C

7. A nurse assesses the vital signs of a patient who is one day postsurgery in which a colostomy was performed. The nurse then uses the data to update the patient plan of care. What are these actions considered? A) initial planning B) comprehensive planning C) on-going planning D) discharge planning

During a health assessment, a 49-year-old woman tells the nurse that she is "just so tired and has been having mood swings and hot flashes." Based on this information, the nurse would conduct a more thorough history and assessment of what body system? A) reproductive B) cardiovascular C) respiratory D) cranial nerves

A

Family members of older adults with limitations from chronic illnesses have many psychological stressors. Which of the following is not considered a change? A) assisting the older family member to function independently B) coping with the physical and economic needs of the ill older adult C) becoming a caregiver, with resultant changes in lifestyle D) assuming a new role, such as providing care for a parent

A

While caring for an elderly man, the nurse observes that his skin is dry and wrinkled, his hair is gray, and he needs glasses to read. Based on these observations, what would the nurse conclude? A) These are normal physiologic changes of aging. B) The observations are not typically found in older adults. C) These are abnormal observations and must be reported. D) Extra teaching will be necessary to prevent complications.

A

An older adult, newly widowed, has been unable to adjust to her change in roles or form new relationships. What is this experience called? A) social isolation B) social ineptness C) ineffective coping D) negativism of aging

A Social isolation is a state of complete or near-complete lack of contact between an individual and society. It differs from loneliness, which reflects a temporary lack of contact with other humans. Social isolation can be an issue for individuals of any age, though symptoms may differ by age group.

The nurse participates in a quality assurance program and reviewing evaluation data from the previous year. Which of the following does the nurse recognize as an example of outcome evaluation?

A 2% reduction in the number of repeat admissions for clients who underwent hip replacement surgery.

A nurse performs presurgical assessments of patients in an ambulatory care center. Which patient would the nurse report to the surgeon as possibly needing surgery to be postponed?

A 43-year-old patient who takes ginko bilboa and an aspirin daily

The nurse working on a medical surgical floor should determine that which of the following clients are at risk for a fall? (Select all that apply.)

A client taking a diuretic A client with advanced alzheimer disease A client with macular degeneration

Which of the following provides the nurse with the most reliable basis on which to choose a nursing diagnosis?

A cluster of several significant cues of data that suggest a particular health problem

Which of the following clients is most likely to face an increased risk of falls due to his or her medication regimen? A) A female client age 77 years who has received a benzodiazepine to minimize her anxiety B) A male client age 79 years whose recent high blood pressure has required a PRN dose of an angiotensin-converting enzyme (ACE) inhibitor C) A woman age 81 years who has required a blood transfusion to treat a gastrointestinal bleed D) A man 90 years of age whose venous ulcer has required the administration of intravenous antibiotics

A female client age 77 years who has received a benzodiazepine to minimize her anxiety

For which of the following clients should the nurse anticipate the need for a pureed diet?

A man whose stroke has resulted in difficulty swallowing

Which of the following nurses most likely is the best communicator?

A nurse who easily developed a rapport with clients.

Which of the following nurses is most likely to care for clients who are trying to resolve identity versus role confusion?

A nurse who provides care in a large junior high school

Which client will have an increased metabolic rate and require nutritional interventions?

A person with a serious infection and fever

The nurse has entered a client's room and observes that the client is hunched over and appears to be breathing rapidly. What type of question should the nurse first implement in this interaction?

A yes/no question

A client with diabetes has impaired sensation in her lower extremities. What education would be necessary to reduce her risk of injury? A) "Always test the temperature of bath water before stepping in." B) "Take your insulin twice a day as we have discussed." C) "Remember to follow your diet so you lose weight this month." D) "Rub lotion on the skin of your legs and feet twice a day."

A) "Always test the temperature of bath water before stepping in."

A girl age 4 years has been admitted to the emergency department after accidently ingesting a cleaning product. Which of the following treatments is most likely appropriate in the immediate treatment of the girl's poisoning? A) Administration of activated charcoal B) Inducing vomiting C) Gastric lavage D) Intravenous rehydration

A) Administration of activated charcoal

The nurse is caring for a client who has prescribed extremity restraints. The nurse is required to document which of the following? A) Alternative measures attempted before applying the restraints B) A verbal order for renewal of the restraints every 48 hours C) Detailed description of the restraint application process D) Type of personal protective equipment (PPE) used by the nurse during restraint application

A) Alternative measures attempted before applying the restraints

Which of the following populations, based on their development stage, would benefit from strategies to prevent falls? Select all that apply. A) Newborns B) Toddlers C) Adolescents D) Adults E) Older Adults

A) Newborns B) Toddlers E) Older Adults

In which of the following situations is it protocol for the nurse to take a patient's vital signs? Select all that apply. A) upon admitting a patient to a hospital B) at a healthcare screening C) when medications are given for a cardiac arrhythmia D) following a diagnostic procedure E) prior to an invasive procedure F) when daily medications are dispensed

A) upon admitting a patient to a hospital B) at a healthcare screening C) when medications are given for a cardiac arrhythmia D) following a diagnostic procedure E) prior to an invasive procedure

According to Havighurst, which of the following are developmental tasks of middle adulthood? A) Accept and adjust to physical changes. B) Maintain a satisfactory occupation. C) Assist children to become responsible adults. D) Maintain social contacts and relationships. E) Relate to one's spouse or partner as a person.. F) Be flexible and adapt to age-related roles.

A, B, C, E

A recently widowed 80-year-old male is dehydrated and is admitted to the hospital for intravenous fluid replacement. During the evening shift, the patient becomes acutely confused. The nurse's best action is to assess the patient for which of the following reversible causes? (Select all that apply.) a. Electrolyte imbalance b. Hypoglycemia c. Drug effects d. Dementia e. Cerebral anoxia

A, B, C, E Delirium, or acute confusional state, is a potentially reversible cognitive impairment that is often due to a physiological event. Physiological causes of delirium can include electrolyte imbalances, cerebral anoxia, hypoglycemia, medications, drug effects, tumors, subdural hematomas, and cerebrovascular infection, infarction, or hemorrhage. Unlike delirium, dementia is a gradual, progressive, irreversible cerebral dysfunction.

Which of the following are physical changes that occur in middle adulthood? Select all that apply. A) Body fat is redistributed. B) The skin is more elastic. C) Cardiac output begins to increase. D) Muscle mass gradually decreases. E) There is a loss of calcium from bones. F) Hormone production increases.

A, D, E

A 71-year-old patient enters the emergency department after falling down stairs in the home. The nurse is conducting a fall history with the patient and his wife. They live in a one-level ranch home. He has had diabetes for over 15 years and experiences some numbness in his feet. He wears bifocal glasses. His blood pressure is stable at 130/70. The patient does not exercise regularly and states that he experiences weakness in his legs when climbing stairs. He is alert, oriented, and able to answer questions clearly. What are the fall risk factors for this patient? (Select all that apply.) A. Impaired vision B. Residence design C. Blood pressure D. Leg weakness E. Exercise history

A, D, E Risk factors for falling include sensory changes such as visual loss, musculoskeletal conditions affecting mobility (in this case weakness), and deconditioning (from lack of exercise). The mere presence of a chronic disease is not a risk factor unless it is a condition such as a neurological disorder that alters mobility or cognitive function. The patient's blood pressure is stable, and there is no report of orthostatic hypotension. A one-floor residence should not pose risks.

Which of the following are examples of common factors that may influence assessment priorities? Select all that apply. A. a patient's diet and exercises program B. a patient's standing in the community C. a patient's ability to pay for services D. a patient's developmental stage E. a patient's need for nursing

A. a patient's diet and exercises program D. a patient's developmental stage E. a patient's need for nursing

A nurse is prioritizing the following patient diagnoses according to Maslow's hierarchy of human needs: (1) Disturbed Body Image (2) Ineffective Airway Clearance (3) Spiritual Distress (4) Impaired Social Interaction Which answer choice below lists the problems in order of highest priority to lowest priority based on Maslow's model? a. 2, 4, 1, 3 b. 3, 1, 4, 2 c. 2, 4, 3, 1 d. 3, 2, 4, 1

A. 2 (physiologic), 4 (love and belonging), 1 (self-esteem), 3 (self-actualization)

Which of the following examples of patient data needs to be validated? Select all that apply. A. A patient has trouble reading an informed consent , but states he does not need glasses B. An elderly patient explains that the black and blue marks on his arms and legs are due to a fall C. A nurse examining a patient with a respiratory infection documents fever and chills D. A patient in a nursing home states that she is unable to eat the food being served. E. A pregnant patient is experiencing contraction that are 2 minutes apart F. Following a MVA, the teenage driver with alcohol on his breath state that he was not drinking

A. A patient has trouble reading an informed consent , but states he does not need glasses B. An elderly patient explains that the black and blue marks on his arms and legs are due to a fall F. Following a MVA, the teenage driver with alcohol on his breath state that he was not drinking

The nurse is completing a health history with the daughter of a newly admitted patient who is confused and agitated. The daughter reports that her mother was diagnosed with Alzheimer's disease 1 year ago but became extremely confused last evening and was hallucinating. She was unable to calm her, and her mother thought she was a stranger. On the basis of this history, the nurse suspects that the patient is experiencing: A. Delirium. B. Depression. C. New-onset dementia. D. Worsening dementia.

A. Delirium. Hallmark characteristics of delirium are acute confusion, hallucinations, and agitation. It is not a new onset of dementia since she already has a diagnosis of Alzheimer's disease and, as dementia worsens, we see a gradual rather than sudden changes in memory usually not accompanied with hallucinations. Depression does not present with acute confusion and agitation.

An acronym RACE is commonly taught as a means for remembering priorities for action during a fire. The "A" in this acronym stands for which of the following?

Activate the fire alarm and notify the appropriate person.

A nurse is using critical pathway methodology for choosing interventions for a patient who is receiving chemotherapy for breast cancer. Which nursing actions are characteristics of this system being used when planning care? Select all that apply. a. The nurse uses a minimal practice standard and is able to alter care to meet the patient's individual needs. b. The nurse uses a binary decision tree for stepwise assessment and intervention. c. The nurse is able to measure the cause-and-effect relationship between pathway and patient outcomes. d. The nurse uses broad, research-based practice recommendations that may or may not have been tested in clinical practice. e. The nurse uses preprinted provider orders used to expedite the order process after a practice standard has been validated through research. f. The nurse uses a decision tree that provides intense specificity and no provider flexibility.

A. The nurse uses a minimal practice standard and is able to alter care to meet the patient's individual needs. C. The nurse is able to measure the cause-and-effect relationship between pathway and patient outcomes.

Which of the following statements accurately describe the impact on nursing of using NIC/NOC standardized languages? Select all that apply. A) They demonstrate the impact that nurses have on the system of healthcare delivery. B) They standardize and define the knowledge base for nursing curricula and practice. C) They limit the number of appropriate nursing intervention to be selected. D) They hinder the teaching of clinical decision making to novice nurses. E) They enable researchers to examine the effectiveness and cost of nursing care. F) They slow the development and use of nursing information systems.

A. They demonstrate the impact that nurses have on the system of healthcare delivery. B. They standardize and define the knowledge base for nursing curricula and practice. E. They enable researchers to examine the effectiveness and cost of nursing care.

In which of the following patients has the order of priorities for nursing diagnoses changed? Select all that apply. A) a patient in a long-term care facility who had a stroke B) a patient who is recovering from a broken leg C) a patient who insists on using the bathroom instead of a bedpan D) a patient who appears confused after taking pain medication E) a pregnant patient whose contractions are progressing as anticipated F) a patient who has wounds that require stitches as well as a concussion

A. a patient in a long-term care facility who had a stroke C. a patient who insists on using the bathroom instead of a bedpan D. a patient who appears confused after taking pain medication F. a patient who has wounds that require stitches as well as a concussion

Which of the following is categorized as a psychomotor outcome? A) Within 2 days of teaching, the patient's wife will demonstrate abdominal dressing change. B) Within 1 week of attending class, the patient will have cut smoking from 20 to 10 cigarettes per day. C) The patient will verbalize understanding of need to continue to take medications as prescribed. D) The patient's skin will remain smooth, moist, and without breakdown or ulceration.

A. Within 2 days of teaching, the patient's wife will demonstrate abdominal dressing change.

What common problem is related to outcome identification and planning? A) failing to involve the patient in the planning process B) collecting sufficient data to establish a database C) stating specific and measurable outcomes based on nursing diagnoses D) writing nursing orders that are clear and resolve the problem

A. failing to involve the patient in the planning process

The nursing diagnosis Impaired Gas Exchange, prioritized by Maslow's hierarchy of basic human needs, is appropriate for what level of needs? A) physiologic B) safety C) love and belonging D) self-actualization

A. physiologic

Which of the following activities related to respiratory health is an example of tertiary health promotion and illness prevention? a) Assisting with lung function testing of a patient to help determine a diagnosis. b) Teaching a patient that "light" cigarettes do not prevent lung disease. c) Advocating politically for more explicit warning labels on cigarette packages. d) Administering a nebulized bronchodilator to a patient who is short of breath.

Administering a nebulized bronchodilator to a patient who is short of breath.

What population is at greatest risk for hypertension? A) Hispanic B) White C) Asian D) African American

African American

A school nurse is teaching a group of adolescents about safe driving. What behaviors should the nurse encourage in order to help prevent motor vehicle accidents? (Select all that apply.)

Always wear a seat belt. Limit the number of other adolescents in the car. Never text while driving. Obey the speed limit.

A diagnosis of chronic stress is supported by which assessment finding? Select all that apply. Amenorrhea Loss of appetite History of panic attacks Insulin-resistant diabetes Recent treatment for blood clots

Amenorrhea History of panic attacks Insulin-resistant diabetes

What nursing organization first legitimized the use of the nursing process? a) American Nurses Association b) State Board of Nursing c) National League for Nursing d) International Council of Nursing

American Nurses Association

A client visits a health care facility with complaints of loss of appetite following a prolonged illness. How should the nurse document the client's condition?

Anorexia

A patient is experiencing hypoxia. Which of the following nursing diagnoses would be appropriate?

Anxiety

The nurse is informed while receiving a nursing report that the client has been hypoxic during the evening shift. Which of the following assessment findings is consistent with hypoxia?

Anxiety, restlessness, confusion, or drowsiness are common signs of hypoxia.

The nurse has just admitted a client with a latex allergy to the medical-surgical nursing floor. Which is the priority nursing intervention?

Apply an allergy-alert identification bracelet on the client.

A nurse is planning care for an adult client with severe hearing impairment that uses sign language and lip reading for communication that has a new diagnosis of cancer. Which nursing action is most appropriate when establishing the plan of care?

Arrange for a sign language interpreter when discussing treatment.

A client has just been taught about lowering cholesterol with diet and exercise. What is the best way to evaluate that the client understands the material? a) Accept silence as client understanding. b) Ask direct questions about the teaching plan. c) Allow the client to discuss personal issues. d) Redirect conversation to the topic.

Ask direct questions about the teaching plan.

A 19-year-old male college basketball player is being evaluated for injuries after a skiing accident. The nurse determines the client has a pulse of 52. What would be the most appropriate way for the nurse to determine the significance of the client's heart rate?

Ask the client if the heart rate is normal to him.

A student takes an adult patient's pulse and counts 20 beats/min. Knowing this is not the normal range for adult pulse, what should the student do next? A. Record the pulse rate on the appropriate vital signs sheet in the chart B. Ask the instructor or a staff nurse to take a pulse C. Discuss this finding during post conference with other students D. wait 4 hours and take the patient's pulse again

Ask the instructor or a staff nurse to take a pulse

The nurse overhears an older client's son talking to her in a very aggressive and violent way. When the nurse walks into the room, the son changes and speaks kindly to his mother and the health care providers. What should the nurse do about this observation?

Ask to examine the client alone in order to speak to her privately.

A nursing student is performing an assessment of a patient on a hospital unit. What component of the patient assessment will allow the student to document data relating to a subjective way of knowing?

Asking the patient to describe the character of her abdominal pain

What should the nurse do for a patient who is experiencing the alarm stage of stress? Assess for a dry mouth Determine the blood pressure Check oxygen saturation level Assess for signs of depression

Assess for a dry mouth

The nurse is palpating the skin of a 30-year old client and documents that when picked up in a fold, the skin fold slowly returns to normal. What would be the next action of the nurse based on this finding?

Assess the client for dehydration.

One hour after receiving pain medication, a postoperative client complains of intense pain. What is the nurse's most appropriate first action? a) Discuss the frequency of pain medication administration with the client. b) Assess the client to determine the cause of the pain. c) Assist the client to reposition and splint the incision. d) Consult with the physician for additional pain medication.

Assess the client to determine the cause of the pain.

The nurse is preparing to administer a blood pressure medication to a client. To ensure the client's safety, what is the priority action for the nurse to take? a) Tell the client to report any side effects experienced. b) Assess the client's blood pressure to determine if the medication is indicated. c) Determine the client's reaction to the medication in the past. d) Ask the client to verbalize the purpose of the medication.

Assess the client's blood pressure to determine if the medication is indicated.

A client has had a left-side mastectomy. How does this affect the blood pressure assessment?

Assessment of blood pressure is impeded

A school-aged child is admitted to the Emergency Room with the diagnosis of a concussion following a collision when playing football. After the collision, the parents state that he was "knocked out" for a few minutes before recognizing his surroundings. What is the priority assessment when the nurse first sees the patient?

Assessment of vital signs and respiratory status

During the inspection of a client's abdomen, the nurse notes that it is visibly distended. The nurse should proceed with the client's abdominal assessment by next performing which of the following? a) Percussion b) Auscultation c) Light palpation d) Deep palpation

Ausculation

A nurse assesses a client for blood pressure. Which technique would be used for this assessment?

Auscultation

The acute care nurse is assessing a newly admitted client's abdomen. Which finding would indicate the need to contact the primary care provider?

Auscultation of a bruit

A mother brings her toddler, age 20 months, to the clinic today for immunizations. She talks about trying to initiate toilet training a few weeks ago, but her son wasn't interested. She decided to put it off for awhile. She told her son he was a good boy and they would try again another time. According to Erik Erikson's theory, what is the likely outcome for the toddler's developmental stage?

Autonomy

What is the most important safety concept that a nurse should include in the teaching plan for a family with a newborn infant in the household?

Avoid stuffed animals and blankets in the crib.

A nurse is teaching a group of middle adults about health promotion. What statement by one of the participants indicates the need for additional teaching? A) "I will make exercise a part of my daily activities." B) "I should eat a diet high in fats but low in fiber." C) "I will begin a smoking cessation program this week." D) "I only have one glass of wine a day with dinner."

B

According to Erikson, the middle adult is in a period of generativity versus stagnation. What happens if developmental tasks are not achieved? A) physical changes are denied B) health needs become a major concern C) motivation to learn is decreased D) awareness of own mortality increases

B

Which of the following statements is true for nursing care of older adults? A) Most older adults are unable to care for themselves independently. B) Most older adults are functional, benefiting from health-oriented interventions. C) Fewer older adults will require nursing care during the 21st century. D) Interventions for older adults are no different from those for young adults.

B

While conducting a health assessment with an older adult, the nurse notices it takes the person longer to answer questions than is usual with younger patients. What should the nurse do? A) Stop asking questions so as not to confuse the patient. B) Slow the pace and allow extra time for answers. C) Realize that the patient has some dementia. D) Ask a family member to answer the questions.

B

A teenager states, "Old people are different. They don't need the same things that young people do." What is this statement an example of? A) racism B) ageism C) indifference D) knowledge

B Ageism (also spelled "agism") is stereotyping and discriminating against individuals or groups on the basis of their age. This may be casual or systematic. The term was coined in 1969 by Robert Neil Butler to describe discrimination against seniors, and patterned on sexism and racism. Butler defined "ageism" as a combination of three connected elements. Among them were prejudicial attitudes towards older people, old age, and the aging process; discriminatory practices against older people; and institutional practices and policies that perpetuate stereotypes about elderly people.

The daughter of an older adult calls the nurse practitioner to report that her mother is becoming very confused after dark. What is this type of confusion named? A) night-time confusion B) sundowning syndrome C) Alzheimer's disease D) cognitive dysfunction

B Sundowning, or sundown syndrome is a neurological phenomenon associated with increased confusion and restlessness in patients with delirium or some form of dementia.

An adolescent has recently had a ring inserted into her navel. Which of the following is the greatest risk facing the adolescent as a result of this activity? A) A scar over the navel B) A local and/or systemic infection C) A greater acceptance by peers D) A strained relationship with parents

B) A local and/or systemic infection

An older adult lives in a facility that provides, housing, group meals, personal care and support, social activities, and minimal healthcare services. What type of facility does this describe? A) nursing home B) assisted living C) accessory apartment D) home modification

B) assisted living assisted living is a housing facility for people with disabilities or for adults who cannot or chose not to live independently. The term is popular in the United States but is similar to a retirement home in the sense that facilities provide a group living environment and typically cater to an elderly population.

A patient's family member is considering having her mother placed in a nursing center. The nurse has talked with the family before and knows that this is a difficult decision. Which of the following criteria does the nurse recommend in choosing a nursing center? (Select all that apply.) A. The center needs to be clean, and rooms should look like a hospital room. B. Adequate staffing is available on all shifts. C. Social activities are available for all residents. D. The center provides three meals daily with a set menu and serving schedule. E. Staff encourage family involvement in care planning and assisting with physical care.

B, C, E Adequate staffing, provision of social activities, and active family involvement are essential. Meals should be high quality with options for what to eat and when it is served. A nursing center should be clean, but it should look like a person's home rather than a hospital.

A nurse is caring for a patient preparing for discharge from the hospital the next day. The patient does not read. His family caregiver will be visiting before discharge. What can the nurse do to facilitate the patient's understanding of his discharge instructions? (Select all that apply.) A. Yell so the patient can hear you. B. Sit facing the patient so he is able to watch your lip movements and facial expressions. C. Present one idea or concept at a time. D. Send a written copy of the instructions home with him and tell him to have the family review them. E. Include the family caregiver in the teaching session.

B, C, E Teaching and communication are more effective with older adults when you sit and face the patient and present one idea or concept at a time. This requires planning. Speaking loudly can distort sound. Speak in a normal tone. Sending instructions is helpful but will not directly facilitate the patient's own understanding. Sharing information with a caregiver provides someone to clarify instructions.

A nurse is participating in a health and wellness event at the local community center. A woman approaches and relates that she is worried that her widowed father is becoming more functionally impaired and may need to move in with her. The nurse inquires about his ability to complete activities of daily living (ADLs). ADLs include independence with: (Select all that apply.) A. Driving. B. Toileting. C. Bathing. D. Daily exercise. E. Eating.

B, C, E ADLs are self-care tasks that measure function and are markers for the ability to live independently. Although driving and daily exercise are important to quality of life and health maintenance, they would not necessarily impact a person's ability to live independently.

During a home health visit a nurse talks with a patient and his family caregiver about the patient's medications. The patient has hypertension and renal disease. Which of the following findings place him at risk for an adverse drug event? (Select all that apply.) A. Taking two medications for hypertension B. Taking a total of eight different medications during the day C. Having one physician who reviews all medications D. Patient's health history of renal disease E. Involvement of the caregiver in helping with medication administration

B, D The patient is at risk for an adverse drug event (ADE) because of polypharmacy and his history of renal disease, which affects drug excretion. Taking two medications for hypertension is common. Having one physician review all medications and involving a family caregiver are desirable and are safety factors for preventing ADEs.

A nurse on a busy surgical unit relies on informal planning to provide appropriate nursing responses to patients in a timely manner. What are examples of this type of planning? Select all that apply. a. A nurse sits down with a patient and prioritizes existing diagnoses. b. A nurse assesses a woman for postpartum depression during routine care. c. A nurse plans interventions for a patient who is diagnosed with epilepsy. d. A busy nurse takes time to speak to a patient who received bad news. e. A nurse reassesses a patient whose PRN pain medication is not working. f. A nurse coordinates the home care of a patient being discharged.

B. A nurse assesses a woman for postpartum depression during routine care. D. A busy nurse takes time to speak to a patient who received bad news. E. A nurse reassesses a patient whose PRN pain medication is not working.

A nurse is assessing an older adult brought to the emergency department following a fall and wrist fracture. She notes that the patient is very thin and unkempt, has a stage 3 pressure ulcer to her coccyx, and has old bruising to the extremities in addition to her new bruises from the fall. She defers all of the questions to her caregiver son who accompanied her to the hospital. The nurse's next step is to: A. Call social services to begin nursing home placement. B. Ask the son to step out of the room so she can complete her assessment. C. Call adult protective services because you suspect elder mistreatment. D. Assess patient's cognitive status.

B. Ask the son to step out of the room so she can complete her assessment. The assessment leads you to suspect elder mistreatment, but the nurse needs more information directly from the patient before calling social services or the adult protective services. She will best get this information by asking the son to leave so she can ask the patient direct questions privately. If the son refuses to leave, this will be another indication that elder mistreatment may be occurring. Cognitive testing will be important but is not the priority.

A nurse conducted an assessment of a new patient who came to the medical clinic. The patient is 82 years old and has had osteoarthritis for 10 years and diabetes mellitus for 20 years. He is alert but becomes easily distracted during the assessment. He recently moved to a new apartment, and his pet beagle died just 2 months ago. He is most likely experiencing: A. Dementia. B. Depression. C. Delirium. D. Hypoglycemic reaction.

B. Depression. Factors that often lead to depression include presence of a chronic disease or a recent change or life event (such as loss). Patients are alert but easily distracted in conversation.

A nurse is caring for an elderly male patient who is receiving fluids for dehydration. Which outcome for this patient is correctly written? a. Offer the patient 60 mL fluid every 2 hours while awake. b. During the next 24-hour period, the patient's fluid intake will total at least 2,000 mL. c. Teach the patient the importance of drinking enough fluids to prevent dehydration by 1/15/15. d. At the next visit, 12/23/15, the patient will know that he should drink at least 3 liters of water per day.

B. During the next 24-hour period, the patient's fluid intake will total at least 2,000 mL.

A nurse has developed a plan of care with nursing interventions designed to meet specific patient outcomes. The outcomes are not met by the time specified in the plan. What should the nurse do now in terms of evaluation? A) Continue to follow the written plan of care. B) Make recommendations for revising the plan of care. C) Ask another healthcare professional to design a plan of care. D) State goal will be met at a later date.

B. Make recommendations for revising the plan of care.

What is true of nursing responsibilities with regard to a physician-initiated intervention (physician's order)? A) Nurses do not carry out physician-initiated interventions. B) Nurses do carry out interventions in response to a physician's order. C) Nurses are responsible for reminding physicians to implement orders. D) Nurses are not legally responsible for these interventions.

B. Nurses do carry out interventions in response to a physician's order.

Which of the following is an example of a well-stated nursing intervention? A) Patient will drink 100 mL of water every 2 hours while awake. B) Offer patient 100 mL of water every 2 hours while awake. C) Offer patient water when he complains of thirst. D) Patient will continue to increase oral intake when awake.

B. Offer patient 100 mL of water every 2 hours while awake.

A nurse is collecting more patient data to confirm a diagnosis of emphysema for a 68-year-old male patient. What type of diagnosis does this intervention seek to confirm? a. Actual b. Possible c. Risk d. Collaborative

B. Possible

A nurse is planning care for a male adolescent patient who is admitted to the hospital for treatment of a drug overdose. Which nursing actions are related to the outcome identification and planning step of the nursing process? Select all that apply: a. The nurse formulates nursing diagnoses. b. The nurse identifies expected patient outcomes. c. The nurse selects evidence-based nursing interventions. d. The nurse explains the nursing care plan to the patient. e. The nurse assesses the patient's mental status. f. The nurse evaluates the patient's outcome achievement.

B. The nurse identifies expected patient outcomes. C. The nurse selects evidence-based nursing interventions. D. The nurse explains the nursing care plan to the patient.

Critical thinking is an essential component in all phases of the nursing process. What question might be used to facilitate critical thinking during outcome identification and planning? A) How do I best cluster these data and cues to identify problems? B) What problems require my immediate attention or that of the team? C) What major defining characteristics are present for a nursing diagnosis? D) How do I document care accurately and legally?

B. What problems require my immediate attention or that of the team?

A nurse is discharging a patient from the hospital. When should discharge planning be initiated? A) at the time of discharge from an acute healthcare setting B) at the time of admission to an acute healthcare setting C) before admission to an acute healthcare setting D) when the patient is at home after acute care

B. at the time of admission to an acute healthcare setting

Which of the following groups of terms best describes a nurse-initiated intervention? A)dependent, physician-ordered, recovery B) autonomous, clinical judgment, patient outcomes C) medical diagnosis, medication administration D) other healthcare providers, skill acquisition

B. autonomous, clinical judgment, patient outcomes

A nurse admits a patient to the hospital's short-stay unit and completes a health history and physical assessment. Using these data, the nurse develops a(n) ___________plan of care, based on _____________ planning? A) intermittent, focused B) comprehensive, initial C) single-use, ongoing D) standard, emergency

B. comprehensive, initial

Which of the following are verbs that are helpful in writing measurable outcomes? Select all that apply. A) know B) define C) hear D) verbalize E) feel F) list

B. define D. verbalize F. list

Nurses identifying outcomes and related nursing interventions must refer to the standards and agency policies for setting priorities, identifying and recording expected patient outcomes, selecting evidence-based nursing interventions, and recording the plan of care. Which of the following are recognized standards? Select all that apply. A) professional physicians' organizations B) state Nurse Practice Acts C) The Joint Commission D) the Agency for Health Care Research and Quality E) the Patient Health Partnership F) the Patient Bill of Rights

B. state Nurse Practice Acts C. The Joint Commission D. the Agency for Health Care Research and Quality

A resident of a long-term care facility refuses to eat until she has had her hair combed and her make-up applied. In this case, what patient need should have priority? A) the need to have nutrition B) the need to feel good about oneself C) the need to live in a safe environment D) the need for love from others

B. the need to feel good about oneself

The nurse is educating an adolescent with asthma on how to use a metered-dose inhaler. Which education point follows recommended guidelines?

Be sure to shake the canister before using it.

A nurse takes a patient's vital signs. Which of the following is considered a vital sign? A) mental status B) visual acuity C) blood pressure D) urinary output

Blood pressure

A patient diagnosed with hypertension uses an automatic cycling blood pressure cuff with audible changing tones. The patient uses relaxation techniques to lower blood pressure and is informed of ongoing success by the tone. This process describes Biofeedback Guided imagery Therapeutic touch Assertiveness training

Biofeedback

Which approach to reducing patient stress is most effective in people with low to moderate hypnotic ability? Meditation Biofeedback Journal keeping Breathing exercises

Biofeedback

An elderly woman who is incontinent of stool following a cerebrovascular accident will have the following nursing diagnosis a) Diarrhea related to tube feedings as evidenced by hyperactive bowel sounds and urgency b) Fecal retention related to loss of sphincter control and diminished spinal cord innervation related to hemiparesis c) Constipation related to physiologic condition involving the deficit in neurologic innervation as evidenced by fecal incontinence d) Bowel incontinence related to loss of sphincter control as evidenced by inability to delay the urge to defecate

Bowel incontinence related to loss of sphincter control as evidenced by inability to delay the urge to defecate

Nurses formulate different types of goals for clients when planning client care. What is considered a psychomotor client goal?

By 18AUG2015, client will demonstrate improved motion in left arm.

Nurses formulate different types of goals for patients when planning patient care. What is considered a psychomotor patient goal?

By 8/18/15, patient will demonstrate improved motion in left arm.

A nurse follows the guidelines for a healthy lifestyle. How can this promote health in others? a) By being a role model for healthy behaviors. b) By not requiring sick days from work. c) By never exposing others to any type of illness. d) By spending less money on food.

By being a role model for healthy behaviors.

A nurse advises a stressed patient to perform meditation to decrease stress levels. How does meditation help in relieving stress? By creating a hypometabolic state of quieting the sympathetic nervous system. By allowing switching from the sympathetic mode to the parasympathetic mode. By dampening the cognitive processes likely to induce stress and anxiety reactions. By eliminating muscle contraction and decreasing the related anxiety levels.

By creating a hypometabolic state of quieting the sympathetic nervous system. Meditation helps to elicit relaxation by creating a hypometabolic state of quieting the sympathetic nervous system. A decreased activation of the sympathetic nervous system reduces the stress responses and promotes relaxation. A relaxation response allows switching from the sympathetic mode to the parasympathetic mode. Techniques like deep breathing exercises help to dampen the cognitive processes likely to induce stress and anxiety reactions. Progressive muscle relaxation helps to eliminate muscle contraction and decrease the related anxiety levels. p. 166

A nurse documents the following assessment on an elderly patient's chart: "dry, thin skin." Which of the following nursing diagnosis would be appropriate for this patient? A) risk for falls B) risk for imbalanced body temperature C) risk for infection D) risk for sedentary lifestyle

C

A nurse says to an older adult who is being cared for at home, "Tell me what your life was like when you were first married." What does this statement encourage the patient to do? A) Explain why he or she has certain emotions. B) Become more introspective and self-focused. C) Practice life review or reminiscence. D) Look backward with regret for undone tasks.

C

Mrs. Ash, an 88-year-old woman who lives alone, has deficits in vision and hearing. Her blood pressure medicine is making her dizzy. What response to these health problems would the home health nurse identify? A) decreased social interaction B) altered consciousness C) risk for accidental injury D) risk for impaired judgment

C

Of the following information collected during a nursing assessment, which are subjective data? A. vomiting, pulse 96 B. respiration 22, blood pressure 130/80 C. nausea, abdominal pain D. pale skin, thick toenails

C

Which aging theory describes a chemical reaction that produces damage to the DNA and cell death? A) genetic theory B) immunity theory C) cross-linkage theory D) free radical theory

C According to the cross‐linkage theory of aging proposed by Johan Björkstein in 1942, aging results from the accumulation of intra‐ and intermolecular covalent bonds between molecules, termed "cross‐links." Over time, these cross‐links result in the alteration of the chemical and biological properties of the cell. Although these changes occur at the level of individual cells, these cellular changes can translate into significant dysfunction of body systems. There are several age‐related manifestations of the accumulation of cellular cross‐links and the resulting cellular dysfunction. For example, cross‐links are associated with the loss of elasticity in skin and muscle tissue, stiffening of blood vessel walls, changes in the lens of the eye, delayed wound healing, and reduced joint mobility in aging individuals.

Which of the following illustrates a common error when writing patient outcomes? A) Patient will drink 100 mL of fluid every 2 hours from 6 a.m. to 9 p.m. B) Patient will demonstrate correct sequence of exercises by next office visit. C) Patient will be less anxious and fearful before and after surgery. D) On discharge, patient will list five symptoms of infection to report.

C.

The nurse sees a 76-year-old woman in the outpatient clinic. She states that she recently started noticing a glare in the lights at home. Her vision is blurred; and she is unable to play cards with her friends, read, or do her needlework. The nurse suspects that the woman may have: A. Presbyopia. B. Presbycusis. C. Cataract(s). D. Depression.

C. Cataract(s). Cataracts normally result in blurred vision, sensitivity to glare, and gradual loss of vision. Presbyopia is a common eye condition resulting in a person having difficulty adjusting to near and far vision. The symptoms are not reflective of depression since her vision affects her ability to interact. She has not chosen to avoid her friends.

A nurse records patient data on a folded card and places it in a central file, where it is easily accessible to staff. Which system of care is this nurse using? A) critical pathways B) case management C) Kardex care plan D) concept map care plan

C. Kardex care plan

Which of the following outcomes is correctly written? A) Abdominal incision will show no signs of infection. B) On discharge, patient will be free of infection. C) On discharge, patient will be able to list five symptoms of infection. D) During home care, nurse will not observe symptoms of infection.

C. On discharge, patient will be able to list five symptoms of infection.

When helping a patient turn in bed, the nurse notices that his heels are reddened and plans to place him on precautions for skin breakdown. This is an example of what type of planning? a. Initial planning b. Standardized planning c. Ongoing planning d. Discharge planning

C. Ongoing planning

The nurse is working with an older adult after an acute hospitalization. The goal is to help this person be more in touch with time, place, and person. Which intervention will likely be most effective? A. Reminiscence B. Validation therapy C. Reality orientation D. Body image interventions

C. Reality orientation Reality orientation is a communication technique that can help restore a sense of reality, improve level of awareness, promote socialization, elevate independent functioning, and minimize confusion.

A nurse performing triage in an emergency room makes assessments of patients using critical thinking skills. Which of the following are critical thinking activities linked to assessments? Select all that apply. A. carrying out a physician's order to intubate a patient B. teaching a novice nurse the principles of triage C. Using the nursing process to diagnose a blocked airway D. interviewing a patient suspected of being a victim of abuse privately E. checking with data supplied by a patient with dementia with the family F. teaching a diabetic patient about the importance of proper foot care

C. Using the nursing process to diagnose a blocked airway D. interviewing a patient suspected of being a victim of abuse privately E. checking with data supplied by a patient with dementia with the family

What name is given to tools that are used to communicate a standardized interdisciplinary plan of care for patients within a case management healthcare delivery system? A) Kardex care plans B) computerized plans of care C) clinical pathways D) student care plans

C. clinical pathways

A dietitian is providing an in-service for the nurses on a medical-surgical unit. During the in-service, she informs the group that there are six classes of nutrients, and three supply the body with energy. What are the three sources of energy?

Carbohydrates, protein, and lipids

A client is about to leave the hospital after having surgery for a fractured left femur. It is now in a plaster cast. The client asks how long before the cast will be dry. The nurse notes on his plan of care a learning outcome stating "Client will verbalize appropriate cast care upon discharge." This represents what type of outcome?

Cognitive

The client identifies three strategies for minimizing leakage of an ileostomy bag. This is an example of what type of outcome?

Cognitive outcome

A nurse teaches a patient a technique for examining negative thoughts and restating them in positive ways. This technique is called Guided imagery Wishful thinking Cognitive reframing Confrontational assertion

Cognitive reframing cognitive reframing calls for changing the viewpoint of a situation and replacing it with another viewpoint that fits the facts but is less negative

Which statement related to the evaluation of outcome attainment for a client is correct?

Collecting data related to outcome attainment requires the nurse to know when to collect the data, based upon established time criteria.

Which of the following group of terms best defines assessing in the nursing process? a) Design a plan of care, implement nursing interventions b) Problem focused, time lapsed, emergency based c) Nurse focused, establishing nursing goals d) Collection, validation, communication of client data

Collection, validation, communication of client data

A nurse performs an assessment on a client who has been admitted to a long-term care facility for physical rehabilitation. What is the term for this type of assessment?

Comprehensive assessment

A nurse caring for an older adult client who has dementia observes another nurse putting restraints on the client without a physician's order. The client is agitated and not cooperating. What would be the best initial action of the first nurse in this situation?

Confront the nurse and explain how this could be dangerous for the client.

The nursing instructor informs a student nurse that a patient she is caring for has a chronic neurologic condition and the condition decreases the patient's peristalsis. What nursing diagnosis is the most likely risk for this patient? a) Diarrhea b) Excessive fluid volume c) Deficient fluid volume d) Constipation

Constipation

The nurse is reviewing a healthcare provider's orders in the electronic health record (EHR) and notices several abbreviations. What is the appropriate nursing action?

Contact healthcare provider to clarify order.

A client comes into the clinic for a routine postoperative visit. While the nurse is assessing the level of pain, the client states that there is occasional discomfort but that pain levels have improved daily since returning home from the hospital. What should the nurse's response be regarding the client's plan of care?

Continue the plan of care.

A student nurse is attempting to improve her communication skills. Which therapeutic communication skill is appropriate?

Control the tone of the voice to avoid hidden messages.

A woman comes to the emergency room with her 2-year-old son. She states he woke up and had a loud barking cough. The child is suffering from:

Croup

A middle adult requests visits by the hospital chaplain and reads the Bible each day while hospitalized for treatment of heart problems. What is the individual illustrating? A) midlife transition B) support of the rights of others C) fear for the future D) trust in spiritual strength

D

A nurse is developing a plan of care for an older adult with chronic heart disease. Which of the following factors must be considered? A) Family members do not need to be as involved in care of the older adult. B) Almost 100% of all older adults have limitations from multiple chronic illnesses. C) Few older adults want to maintain their health and independence. D) Medications, hospitalizations, and medical supplies increase economic difficulties.

D

A nurse is providing care to an older adult at home after major abdominal surgery. Which of the following nursing diagnoses would most likely be appropriate? A) Adult Failure to Thrive B) Anticipatory Grieving C) Impaired Memory D) Risk for Infection

D

According to the free radical theory of aging, what substance is affected by aging and causes damage? A) carbohydrates B) proteins C) water D) lipids

D

Which of the following statements best describes the relationship between nursing diagnosis and medical diagnosis? A. The nursing diagnosis confirms the medical diagnosis B. The nursing diagnosis duplicates the medical diagnosis C. There is no relationship between nursing and medical diagnosis D. The nursing diagnosis is based on patient response to the medical diagnosis

D

Which of the following statements is true of the older adult population? A) Old age begins at 65 years of age. B) Most older adults live in nursing homes. C) Older adults are not interested in sex. D) Incontinence is not a part of aging.

D

In what situation would the use of side rails not be considered a restraint? A) The nurse keeps them raised at all times. B) The institution's policies mandate using side rails. C) A visitor requests their use. D) A client requests they be up at night.

D) A client requests they be up at night.

Which of the following people has the greatest risk for accidental injury? A) An infant just learning to crawl B) An older adult who walks two miles a day C) An athlete who exercises on a regular basis D) A worker who operates industrial machines

D) A worker who operates industrial machines

An emergency room nurse is assessing a toddler with multiple bruises and burns. The nurse suspects the toddler has been abused. What is legally required of the nurse? A) Nothing; the nurse has no control over the toddler's home. B) Refer the caregivers of the toddler to a home health nurse. C) Verbally confront the caregivers about the suspicions. D) Report suspicions about the abuse to proper authorities

D) Report suspicions about the abuse to proper authorities

A client is very anxious and states, "I am so stressed." Why do these factors affect the client's safety? A) Stress increases retention of information B) Stress affects interpersonal relationships C) Stress increases concern about hazards D) Stress tends to narrow the attention span

D) Stress tends to narrow the attention span

The nurse is completing an admission assessment with an 80-year old man who experienced a hip fracture following a fall. He is alert, lives alone, and has very poor hygiene. He reports a 20-pound weight loss in the last 6 months following his wife's death, as well as estrangement from his only child. He admits to falls before this most recent fall. What should the nurse suspect? A. Dementia. B. Elder abuse. C. Delirium. D. Alcohol abuse.

D. Alcohol abuse. Hallmarks of alcohol abuse include frequent falling, self-neglect, and poor nutrition, which could result in weight loss and may accompany depression and loss.

Sexuality is maintained throughout our lives. Which of the following answers best explains sexuality in an older adult? A. When the sexual partner passes away, the survivor no longer feels sexual. B. A decrease in an older adult's libido occurs. C. Any outward expression of sexuality suggests that the older adult is having a developmental problem. D. All older adults, whether healthy or frail, need to express sexual feelings.

D. All older adults, whether healthy or frail, need to express sexual feelings. Sexuality is normal throughout the life span, and older adults need to be able to express their sexual feelings.

A nurse is identifying outcomes for a patient who has a leg ulcer related to diabetes. An example of an affective outcome for this patient is: a. Within 1 day after teaching, the patient will list three benefits of continuing to apply moist compresses to leg ulcer after discharge. b. By 6/12/15, the patient will correctly demonstrate application of wet-to-dry dressing on leg ulcer. c. By 6/19/15, the patient's ulcer will begin to show signs of healing (e.g., size shrinks from 3″ to 2.5″). d. By 6/12/15, the patient will verbalize valuing health sufficiently to practice new health behaviors to prevent recurrence of leg ulcer.

D. By 6/12/15, the patient will verbalize valuing health sufficiently to practice new health behaviors to prevent recurrence of leg ulcer.

Older adults frequently experience a change in sexual activity. Which best explains this change? A. The need to touch and be touched is decreased. B. The sexual preferences of older adults are not as diverse. C. Physical changes usually do not affect sexual functioning. D. Frequency and opportunities for sexual activity may decline.

D. Frequency and opportunities for sexual activity may decline. As a result of loss of a loved one or a chronic illness in themselves or their partner, opportunities for sexual activity may decline. Aging does not change the need for touch, and older adults are diverse.

A 63-year-old patient is retiring from his job at an accounting firm where he was in a management role for the past 20 years. He has been with the same company for 42 years and was a dedicated employee. His wife is a homemaker. She raised their five children, babysits for her grandchildren as needed, and belongs to numerous church committees. What are the major concerns for this patient? (Select all that apply.) A. The loss of his work role B. The risk of social isolation C. A determination if the wife will need to start working D. How the wife expects household tasks to be divided in the home in retirement E. The age the patient chose to retire

D. How the wife expects household tasks to be divided in the home in retirement The psychosocial stresses of retirement are usually related to role changes with a spouse or within the family and to loss of the work role. Often there are new expectations of the retired person. This patient is not likely to become socially isolated because of the size of the family. Whether the wife will have to work is not a major concern at this time nor is the age of the patient.

A nursing student is caring for a 78-year-old patient with multiple sclerosis. The patient has had an indwelling Foley catheter in for 3 days. Eight hours ago the patient's temperature was 37.1° C (98.8° F). The student reports her recent assessment to the registered nurse (RN): the patient's temperature is 37.2° C (99° F); the Foley catheter is still in place, draining dark urine; and the patient is uncertain what time of day it is. From what the RN knows about presentation of symptoms in older adults, what should he recommend first? A. Tell the student that temporary confusion is normal and simply requires reorientation B. Tell the student to increase the patient's fluid intake since the urine is concentrated C. Tell the student that her assessment findings are normal for an older adult. D. Tell the student that he will notify the patient's health care provider of the findings and recommend a urine culture

D. Tell the student that he will notify the patient's health care provider of the findings and recommend a urine culture The patient may have subtle symptoms of a urinary tract infection, as evidenced by a slight increase in body temperature, development of confusion, and the dark-colored urine. Temporary confusion is not a normal condition in older adults. Increasing the fluid intake is acceptable but not a recommendation for the set of symptoms the patient presents. The presenting set of symptoms is not normal.

Which of the following types of care plans is most likely to enable the nurse to take a holistic view of the patient's situation? A) Kardex B) case management C) critical pathways D) concept map care plan

D. concept map care plan

A father runs into the emergency room with his 18-month-old son in his arms. The father screams, "Help, he is not breathing!" The nursing diagnosis of Impaired Gas Exchange is what level of priority diagnosis? A) no priority B) low priority C) medium priority D) high priority

D. high priority

What is the primary purpose of the outcome identification and planning step of the nursing process? A) to collect and analyze data to establish a database B) to interpret and analyze data to identify health problems C) to write appropriate patient-centered nursing diagnoses D) to design a plan of care for and with the patient

D. to design a plan of care for and with the patient

Which item would alert the home care nurse to a safety hazard threatening a young child?

Dangling blind cords

A patient's recent diagnosis of colorectal cancer has required a hemicolectomy (removal of part of the bowel) and the creation of a colostomy. The nurse would recognize that the patient's stoma is healthy when it appears: a) dark pink and moist. b) red and dry. c) dark or purple-blue. d) off-white or pale pink.

Dark pink and moist

An effective stress-reduction technique a nurse might teach an individual with performance anxiety is Assertiveness Journal keeping Deep breathing Restructuring and setting priorities

Deep breathing

The nurse is performing a routine assessment of a male client who has an artificial arm as a result of a small plane crash many years earlier. How should the nurse best understand this client's health?

Despite the loss of his limb, the client may consider himself to be healthy.

The nurse manager on an orthopedic unit has determined that the nurses are not keeping the nursing diagnoses up-to-date on client care plans and, in turn, are not using the plan of care. What is a feasible approach to correcting this problem?

Develop a process for periodic review of care plans that focuses on deleting and updating the nursing diagnoses.

During the initial assessment of a 30-year-old patient who has sought care for the first time, the nurse notes that the patient smokes cigarettes regularly. Which of the following nursing actions best demonstrates the principles of primary health promotion? a) Dialoguing with the patient about his or her history of smoking and reasons for smoking and discussing the health implications. b) Performing auscultation and percussion of the patient's anterior and posterior chest and measuring the patient's oxygen saturation. c) Teaching the patient about the relationship between smoking and lung cancer and chronic obstructive pulmonary disease (COPD). d) Referring the patient to a respiratory therapist for lung function testing.

Dialoguing with the patient about his or her history of smoking and reasons for smoking and discussing the health implications.

Which of the following symptoms are known side effects of antibiotics? a) Fecal impaction b) Abdominal bloating c) Diarrhea d) Constipation

Diarrhea

A nurse is caring for a client with constipation. The incidence of constipation tends to be high among clients taking which of the following diets? a) A diet consisting of whole grains, seeds, and nuts b) A diet lacking in fruits and vegetables c) A diet lacking in meat and poultry products d) A diet lacking in glucose and water

Diet lacking in fruits and vegs

A nurse is treating an adolescent who swallowed drain cleaner on a dare from his classmates. Which interventions would the nurse perform related to this situation? (Select all that apply.)

Dilute with milk or water. Contact physician or primary care provider.

The nurse is preparing to auscultate the bowel sounds of a patient with a nasogastric tube in place to low intermittent suction. How shall the nurse approach the assessment of bowel sounds and manage the nasogastric tube? a) Disconnect the nasogastric tube from the suction for 1 hour prior to the assessment of bowel sounds. b) Apply continuous suction to the nasogastric tube during assessment of bowel sounds. c) Disconnect the nasogastric tube from suction during the assessment of bowel sounds. d) Allow the low intermittent suction to continue during the assessment of bowel sounds.

Disconnect the nasogastric tube from suction during the assessment of bowel sounds.

The nursing instructor has given an assignment to a group of nurse practitioner students. They are to break into groups of four and complete a health-promotion teaching project, then present a report back to their fellow students. What project is the best example of health-promotion teaching?

Discussing the importance of preventing sexually transmitted disease to a group of 12th-grade students

What is the psychological state that results in anxiety, depression, confusion, helplessness, hopelessness, and fatigue? Distress Eustress Fight-or-flight response General adaption syndrome

Distress Distress is the result of a negative perception toward a stressor such as a death in the family, financial overload, and school or work demands. Eustress is a positive, beneficial energy that motivates and results in feelings of happiness, hopefulness, and purposeful movement. Fight-or-flight response is a pathway of the response to stress. General adaption syndrome is a theory that shows that different types of stressors bring about different patterns of responses and that it is the degree of stress that is important. p. 159

Which of the following actions should the nurse take during the evaluation phase of the nursing process?

Document improved pain after pain medication administered

The nurse has administered pain medication to a client with a fractured femur. One hour later, the client reports relief of pain. What is the nurse's next action? a) Plan to decrease the pain medication next time. b) Document the effectiveness of the intervention. c) Instruct the client to use imaging and slow breathing. d) Instruct the client to wait as long as possible to ask for medication.

Document the effectiveness of the intervention.

A 55-year-old client visits a health care facility for a scheduled physical assessment. During the assessment, the client complains of difficulty breathing. What suggestion could the nurse make to improve the client's respiratory function in this case?

Drink liberal amounts of fluids.

A child is playing soccer and is involved in a head collision with another player. Which assessment findings should the nurse be alert to that may indicate a concussion? (Select all that apply.)

Drowsiness Headache Vomiting

The nurse manager is holding a staff meeting and indicates that the unit is looking at a 3% budget cut for the coming year. The nurse manager asks the staff what they see as priorities for the unit, and solicits suggestions from the staff as to what budget areas might be reduced. Which standard for establishing and sustaining healthy work environments does this action represent?

Effective decision making

The nurse is caring for a man 79 years of age who was admitted for a hip replacement. He visits with the nurse every chance he gets; he seems lonely. He tells the nurse his wife died two years ago. He has children in nearby cities and has nine grandchildren. He says his life has been good and he is content, even though he misses his wife. The nurse assesses him for developmental issues and knows that he fits well into which of Erikson's stages of development?

Ego integrity versus despair

The nurse caring for a client for several days has assessed that he has been eating poorly during his hospitalization. Which nursing measure should the nurse implement to assist the client in improving his nutritional intake?

Encourage his daughter to prepare food at home and bring it to the client.

A nurse is preparing to assess a client with abdominal pain. What should the nurse do when preparing the client for assessment?

Explain the assessment procedure to the client.

When assessing a client's nonverbal communication, the nurse will assess which characteristic as the most expressive part of the body?

Facial Expressions

A patient who is postoperative day 1 has rung the call light twice during the nurse's shift in order to request assistance transferring to a bedside commode. In both cases, however, the patient has been unable to defecate. In light of the fact that the patient's last bowel movement was the morning of surgery, what action should the nurse first take? a) Obtain a diet change order to increase the amount of fiber in the patient's meals b) Facilitate a more private setting, such as assisting the patient to a bathroom c) Administer a normal saline enema after obtaining the relevant order d) Position the patient on their side and administer a glycerin suppository

Facilitate a more private setting, such as assisting the patient to a bathroom

Priority setting is based on the information obtained during reassessment. Priority setting is used to rank nursing diagnoses. Each of the following contributes to priority setting except which of the following?

Finances of the client

Which of the following are signs and symptoms of poor nutritional status?

Flaky facial skin, facial edema, pale skin color

While caring for an infant who is breast-fed, the nurse assesses the characteristics of the stools. What stool characteristics are expected in breast-fed infants? a) Golden yellow and loose b) Yellow-brown and pasty c) Dark brown and firm d) Green and mucusy

Golden yellow and loose

One effect of stress can be attributed to the stimulation of the hypothalamus-pituitary-adrenal cortex, causing a short-term increase in Heart rate Triglycerides Gluconeogenesis Brain norepinephrine

Gluconeogenesis

Which interventions will the nurse include in the plan of care of a patient diagnosed with generalized anxiety syndrome (GAS) resulting from stress? Select all that apply. Getting up 30 minutes later each morning. Going to sleep 30 minutes earlier than usual. Listening to soothing music to increase relaxation. Including 30 minutes of exercise into your daily routine three days a week. Including some form of exercise about 30 minutes before going to bed.

Going to sleep 30 minutes earlier than usual. Listening to soothing music to increase relaxation. Including 30 minutes of exercise into your daily routine three days a week.

Of the following statements, which is most true of health and illness? a) Health and illness are the same for all people. b) Health and illness are individually defined by each person. c) People with acute illnesses are actually healthy. d) People with chronic illnesses have poor health beliefs.

Health and illness are individually defined by each person.

A nursing instructor is working with a class of first semester nursing students. The instructor explains the interrelatedness of health and wellness. What would be the best definition the nursing instructor could give of health?

Health is a state of complete physical, mental, and social well-being, not merely the absence of disease or infirmity.

Which of the following actions are included in the planning process when a nurse is caring for an elderly client with AIDS?

Identify measurable goals or outcomes

The nursing student is visiting a middle school with an assignment to observe and visit with students while walking around with the school nurse. Then nursing student is interested to learn that some students seem to be rebelling against authority figures like teachers and parents. The nursing student recalls that, according to Erikson's theory, this is not abnormal behavior. To which stage of Erikson's theory does this behavior belong?

Identity versus role confusion

A nursing student is visiting a third-grade class to observe growth and development in action and does assessments on the children. They are learning to think logically and to classify and relate objects and ideas. According to Erikson, in what developmental stage are they?

Industry versus inferiority

A client is being admitted from the emergency room with complaints of shortness of breath, wheezing, and coughing. Which of the following would the nurse as an appropriate nursing diagnosis?

Ineffective airway clearance

A client with diabetes mellitus has been admitted to the hospital in diabetic ketoacidosis. During the admission assessment of the client, the nurse learns that the client is not following the prescribed therapeutic regimen. The client states, "I don't really have diabetes. My doctor overreacts." What is the most appropriate diagnosis for this client's health problem? a) Ineffective health maintenance related to client's denial of illness b) Risk for injury related to client's mismanagement of disease c) Ineffective coping related to client's inability to manage the diabetic regimen d) Risk for unstable blood glucose related to client's reluctance to manage the diabetic regimen

Ineffective health maintenance related to client's denial of illness

The nurse is preparing a client with a bowel obstruction for emergency surgery. Of the following interventions, which has the highest priority? a) Inform the client what to expect after the surgery. b) Discuss discharge plans with the client. c) Instruct the client and family in wound care. d) Teach the client about dietary restrictions during recovery.

Inform the client what to expect after the surgery.

The unlicensed personnel tells the nurse that a client is very confused and trying to get out of bed without assistance. What is the appropriate action by the nurse?

Initiate use of a bed alarm.

The nurse sees a client age 4 years and her mother at the pediatric clinic. The client is here for immunizations. She is bubbly and talkative, and tells you about her dance lessons and her new baby brother. Her mother tells you she encourages her daughter to try new experiences and enjoy life. You know that according to Erikson's developmental theory, the client is well on the way to achieving which of the following? Initiative

Initiative

A nurse is educating a postoperative client on how to use an incentive spirometer. Which of the following is an accurate step that should be included in the teaching plan?

Instruct the client to inhale slowly and as deeply as possible through the mouthpiece, without using the nose.

A nurse manager notes an increase in the frequency of client falls during the last month. To promote a positive working environment, how would the nurse manager most effectively deal with this problem?

Investigate the circumstances that contributed to client falls.

Which of the following medications causes constipation? a) Iron supplements b) Aspirin c) Dulcolax d) Magnesium antacids

Iron supplements

Then nurse is caring for single, professional woman age 29 years, who was admitted with a severe gall bladder attack. The nurse visits with her and performs an assessment. The client is not married and fears a committed relationship because of a bad experience some years ago. The nurse knows that, according to Erikson's developmental theory, Judith is in danger of which of the following?

Isolation

The nurse is participating in a quality improvement program at the hospital. What are major premises of a quality-improvement program? Select all that apply.

It determines whether nursing standards are being upheld. Some governmental agencies mandate quality improvement programs. It focuses on processes rather than individuals. Its focus is on ensuring excellence in care.

A nurse is caring for a client with chronic obstructive pulmonary disease (COPD). The nurse explains to the client that COPD is a chronic disease. Why is COPD considered a chronic disease? a) It takes a long time to cure. b) It is a sequela of acute illness. c) It persists for a long time. d) It has a gradual onset and lasts for a long time.

It has a gradual onset and lasts for a long time.

The school nurse is teaching growth and development in a health class. What should the nurse teach the students about development?

It is the process of ongoing change

Which teaching about a flowmeter is important for the nurse to provide to a client using oxygen?

It regulates the amount of oxygen received.

A client has been prescribed a clear liquid diet. What food or fluids will be served?

Jell-O, carbonated beverages, apple juice

A patient's last bowel movement was 4 days ago and oral laxatives and dietary changes have failed to prompt a bowel movement. How should the nurse position the patient in anticipation of administering a cleansing enema? a) Right side-lying b) Prone c) Left side-lying d) Supine

Left side-lying

A nurse is preparing to provide care to a client who is receiving radiation therapy for cancer in which the radiation source will be kept in the client's room. Which of the following would be most important for the nurse to do?

Limit own exposure to radiation for the minimum time

Mr. Sanchez is a 56-year-old Mexican American who has a diagnosis of heart failure. The nurse's morning lung assessment of the client reveals crackles in the mid to lower lungs and respiratory rate of 32. The nurse notices that the client is restless, and his skin has an ashen appearance. Which nursing action is the priority intervention?

Measure the pulse oximetry.

The nurse is caring for a client with malnutrition due to protein deficiency. Which of the following foods should be included in this client's diet?

Meats

A nurse gives a speech on nutrition to a group of pregnant women. What is the speech itself known as?

Message

Which of the following is the primary source of heat in the body? A) hormones B) metabolism C) blood circulation D) muscles

Metabolism

A group of students is reviewing the various levels of illness prevention. The students demonstrate understanding when they identify which of the following as a goal of tertiary prevention of illness?

Minimizing complications

The nurse is caring for Mr. H., a 35-year-old man who is hospitalized following a motorcycle accident. He has a traumatic brain injury. The nurse is working with Mr. H. on self-care behaviors. The following would help the nurse to assess the success of the nursing interventions except which of the following?

Model self-care behaviors for the client.

A client has come into the clinic for a postoperative visit. The client states that the postoperative pain continues to be 6 on a 10-point rating scale. The nurse evaluates the patient and the current plan of care. Based on the information provided by the client, the nurse should do which of the following?

Modify the plan of care

A nurse is providing care to a client who has undergone a colonoscopy. Which of the following would be most appropriate for the nurse to do after the procedure? a) Limit oral fluid intake b) Monitor for rectal bleeding c) Avoid giving solid food d) Administer a laxative to the client

Monitor for rectal bleeding

A school nurse is providing information to a group of older adults during Fire Prevention Week. Which statement is correct regarding fires in the home?

Most people who die in house fires die of smoke inhalation, rather than burns.

The nurse is caring for a 48-year-old male patient with a new colostomy. Which patient goal for Mr. Conner is written correctly?

Mr. Conner will demonstrate proper care of stoma by 3/29/15.

A nurse is caring for an asthmatic client who requires a low concentration of oxygen. Which of the following delivery devices should the nurse use in order to administer oxygen to the client?

Nasal cannula

A patient with a diagnosis of stage II Alzheimer disease also has a history of chronic obstructive pulmonary disease (COPD). Which of the following medication delivery systems is most appropriate for this patient?

Nebulizer

A nurse is caring for a young adult female client who has a folic acid defiency. When teaching the client about this condition, the nurse would include a discussion about the client's increased risk for which of the following?

Neural tube deficits in the fetus

A patient in a physician's office has a single blood pressure (BP) reading of 150/92. Should the patient be taught about hypertension? A) It depends on the time of day the BP was taken. B) It depends on whether the patient is male or female. C) No, a single BP reading should not be used. D) Yes, this reading is high enough to be significant.

No, a single BP reading should not be used.

Quality assurance programs are important for ensuring quality nursing care. Which of the following situations need to be reported to the nurse manager? Select all that apply.

Nurse assesses client after sneezing into hand. Nurse administers medications to wrong client. Nurse delays answering call lights to an abusive client. Nurse refuses to provide care to a client with HIV.

A nurse is evaluating the plan of care for the client under her care. Which of the following problems might the nurse note that is associated with the implementation phase of the plan of care?

Nurses are not aware of client priorities and the plan of care.

What is true of nursing responsibilities with regard to a physician-initiated intervention (physician's order)?

Nurses do carry out interventions in response to a physician's order.

A nursing student is observing in a pediatric clinic. A grandmother brings an infant age 2 months to be seen. The infant has failed to gain the expected amount of weight and looks unwell. The nursing student wonders if this may be a "failure to thrive" baby. Which one of the following has been linked to "failure to thrive" babies?

Nutritional deprivation

A nurse is caring for a client who is receiving an intravenous therapy through an IV pump. Which intervention should the nurse implement to ensure electrical safety?

Obtain a three-prong grounded plug adapter.

A nurse assesses the vital signs of a patient who is one day postsurgery in which a colostomy was performed. The nurse then uses the data to update the patient plan of care. What are these actions considered?

Ongoing Planning

A client was admitted two days ago with sepsis. The nurse updates the client's care plan based upon improvements in his condition. This is an example of which type of planning?

Ongoing planning

When caring for a psychiatric patient, a formal contract is made with the patient during which phase of the nurse-patient relationship?

Orientation phase

A client has been reporting persistent headaches. Which is an example of subjective data?

Pain is 4 out of 10 on a pain scale.

When a nurse notices the client is in pain and needs to learn to walk on crutches, which outcome identification is the priority? a) Capillary refill b) Safe walking c) Pain management d) Crutch walking

Pain management

Nurses in all venues have an obligation to provide health education to patients. Who is ultimately responsible for maintaining and promoting health?

Patients

Nurses are involved in many types of evaluation. All of the following are activities that are related to evaluation, but which of the following is the priority concern for nurses?

Patients and their care

A group of nurses of the orthopedic floor of a hospital wish to improve their clinical performance. The nurse manager suggests a program in which the nurses will evaluate each other and provide feedback for improved performance. The nurses know that this program is termed which of the following?

Peer review

A nurse is caring for a client with an abdominal injury at a health care facility. The client tells the nurse that he passed blood-stained stool. Which of the following nursing actions is appropriate when a client reports blood in the stool? a) Save a sample of the stool in a container. b) Report about the stool to the physician. c) Send the stool sample to the laboratory. d) Perform a screening test on stool samples.

Perform a screening test on stool samples.

The nurse is preparing to assess a rectal temperature on an adult client. What is the appropriate nursing intervention?

Provide privacy for the client.

Prior to the first postoperative visit post gastrectomy, the client will have a weight loss of 10 lb (4.50 kg). This is an example of which type of evaluative statement?

Physical changes

The mother brings her infant into the clinic. The infant is two months old and has not been gaining weight appropriately. The outcome statement on the plan of care states "The infant will double birth weight by 6 months of age." This is an example of what type of outcome statement?

Physical changes

A client in a long-term care facility has become increasingly unsteady. The nurses are worried that the client will climb out of bed and fall. Which of the following measures would be a high priority recommendation for this client?

Placing the client in a bed with a bed alarm

A nurse is reviewing the plan of care for a client. What might the nurse identify are problems related to the planning phase of the nursing process? Select all that apply.

Plan of care only contains standard knowledge that most nurses would do if there was a plan of care. Long-term goal is vague. Outcomes are incorrectly developed. Nursing orders are superficial.

A hospitalized client has been NPO with only intravenous fluid intake for a prolonged period. What assessments might indicate protein-calorie malnutrition?

Poor wound healing, apathy, edema

Mr. Ames, age 84, has just been admitted to the hospital for the treatment of pneumonia. In addition to this diagnosis, Mr. Ames also has stage II Alzheimer's disease and is disoriented to place and time. As the night has progressed, he has become increasingly agitated, pulling out his intravenous catheter and wandering throughout the unit. He has become more agitated as the nurses have attempted to reorient and redirect him. Which of the following interventions should the nurses perform?

Position Mr. Ames' bed closer to the nurses' station and perform an assessment

A nurse is giving a talk to a local community group on the importance of proper diet and regular exercise. This is an example of which type of health promotion?

Primary

A nurse is immunizing children against measles. This is an example of what level of preventive care?

Primary

The nurse's community outreach class is giving a presentation on seat belts and child safety seats at the local firehouse every weekend in October. What level(s) of health promotion is this an example of?

Primary

The nurse participates in a quality assurance program and reviews evaluation data for the previous month. The data indicates a nursing plan was developed within 8 hours of admission for 97% of all admissions. The nurse recognizes this as which type of evaluation?

Process evaluation

A nurse is educating a home care client on how to do pursed-lip breathing. What is the therapeutic effect of this procedure?

Prolonging expiration to reduce airway resistance

A nurse is feeding a patient who is experiencing dysphagia. Which nursing intervention would the nurse initiate for this patient?

Provide a thirty-minute rest period prior to mealtime

The Joint Commission is conducting an accreditation visit at the hospital. What is the focus of the evaluation being conducted?

Quality assurance

The nurse manager observes one of the unit nurses failing to was her hands upon entering a client room. Hospital protocol is washing hands before and after entering a client room. The nurse manager knows that this is an example of:

Quality by inspection

A nurse just reported to the oncoming shift that she had failed to do an ordered dressing change. She reported to the nurse manager that this was the second time this week she had not had time to do the dressing change. The nurse manager recognized that the nurse normally was very punctual and was known to provide good care for her clients however the unit census had been very high on this particular week. The nurse manager knows that quality care must be provided and reports this occurrence as what type of quality approach?

Quality by opportunity

Over the past few weeks, a client in a long-term care facility has become increasingly unsteady. The nurses are worried that the client will climb out of bed and fall. Which of the following measures does not comply with a least restraint policy?

Raising all side rails while the client is in bed

A nurse is taking a patient's temperature and wants the most accurate measurement, based on core body temperature. What site should be used? A) rectal B) oral C) axillary D) forehead

Rectal

The nurse is providing discharge teaching to the family of an older adult client. Which teaching will the nurse include to decrease the risk for electric shock?

Refrain from using extension cords.

The nurse is seeing a male client 6 years of age in the clinic. As the nurse is measuring height and weight for the client, his mother comments on how coordinated he is becoming. The nurse knows that this increase in coordination is which of the following?

Related to development

The physician directs the nurse to use oropharyngeal suctioning to remove liquid secretions for a client with a respiratory problem. What is oropharyngeal suctioning?

Removing secretions from the throat through an orally inserted catheter

The nurse is admitting a client to a medical-surgical unit who states, "If someone brings balloons to me, I might have trouble breathing." What is the appropriate nursing action?

Replace common healthcare items with latex-free equipment.

A nurse walks into a patient's room and finds him having difficulty breathing and complaining of chest pain. He has bradycardia and hypotension. What should the nurse do next? A) Take vital signs again in 15 to 30 minutes. B) Document the data and report it later. C) Ask the patient if he is anxious or afraid. D) Report findings to the physician immediately.

Report findings to the physician immediately.

What structural changes to the respiratory system should a nurse observe when caring for older adults? a) Increased mouth breathing and snoring b) Diminished coughing and gag reflexes c) Increased use of accessory muscles for breathing d) Respiratory muscles becomes weaker

Respiratory muscles becomes weaker

Which action should the nurse take when client data indicate that the stated goals have not been achieved?

Review each preceding step of the nursing process.

Nursing interventions for the client after prostate surgery include assisting the client to ambulate to the bathroom. The nurse concludes that the client no longer requires assistance. What is the nurse's best action? a) Continue assisting the client to the bathroom to ensure the client's safety. b) Instruct the client's family to assist the client to ambulate to the bathroom. c) Consult with the physical therapist to determine the client's ability. d) Revise the care plan to allow the client to ambulate to the bathroom independently.

Revise the care plan to allow the client to ambulate to the bathroom independently.

A nurse is caring for a client who is recovering from stroke. Which of the following would the nurse perform in the evaluation phase?

Revise the plan of care

According to Maslow's hierarchy of needs, which nursing diagnosis has the lowest priority for a client admitted to the intensive care unit with a diagnosis of congestive heart failure? a) Ineffective coping b) Impaired urinary elimination c) Risk for body image disturbance d) Ineffective airway clearance

Risk for body image disturbance

Which of the following nursing diagnoses has the highest priority when caring for an older adult client with Alzheimer's disease?

Risk for injury

A client experiencing symptoms of cold is referred to the specialist for diagnosis and consultation. Consultation and diagnostic tests are included in which level of the health care system?

Secondary

The client is admitted with a gastrointestinal bleed (GI). The physician ordered a colonoscopy. Which level of care encompasses this procedure?

Secondary

A nurse is writing an initial plan of care for a client with a rare condition. The nurse has little experience with the condition. What action by the nurse will result in the best plan of care?

Seek research about the disorder.

The client recently immigrated from Mumbai, India. The client was just admitted to the nurse's unit postoperatively following gallstone removal. The client does not speak the dominant language. When using the hospital's interpretive services, which is most important?

Speak directly to the client.

The nurse is caring for a patient who has dysphagia and is unable to eat independently. While assisting the patient in eating, which of the following actions is most appropriate for the nurse?

Speak to the patient, but reduce the number of distractions while patient is eating.

What is the term used to describe the psychological or physical stimuli that are incompatible with the current functioning of an individual and require adaptation in the human body? Endorphins Cytokines Stressors Hormones

Stressors

The nurse is reading another nurse's notes that were recorded in the electronic health record (EHR) during the previous shift. What is the appropriate nursing action when numerous unapproved abbreviations are noticed in the previous nurse's notes?

Suggest to the nurse manager that an in-service on abbreviation use would be helpful.

The nurse is teaching the caregiver of an infant about safety. Which teaching will the nurse include?

Supervise your child on the changing table.

What can a nurse ask a patient to do before suctioning to prevent hypoxemia?

Take several deep breaths.

The nurse is caring for an 80-year-old patient who was admitted to the hospital in a confused and dehydrated state. After the patient got out of bed and fell, restraints were applied. She began to fight and was rapidly becoming exhausted. She has black-and-blue marks on her wrists from the restraints. What would be the most appropriate nursing intervention for this patient?

Take the restraints off, stay with her, and talk gently to her.

A client who was admitted to the acute care unit with angina pectoris is no longer having chest pain. Based on this assessment, what does the nurse decide to do with the plan of care for chest pain?

Terminate the plan of care related to the nursing diagnosis of chest pain

A nurse refers an HIV-positive client to a local support group. This is an example of what level of preventive care?

Tertiary

A nurse is caring for a client with complaints of chest pain. Which of the following test results would indicate whether the client is at risk for cardiac disease?

Test results for dyslipidemia

The nurse needs to plan the interventions necessary to reduce fall risks for the older adult clients at her facility. Which is the strongest indicator that a client is at risk for falls?

The client has fallen before.

Which client outcome is an example of a cognitive outcome?

The client identifies three strategies for minimizing leakage of an ileostomy bag.

A male client has been recently diagnosed with diabetes after receiving emergency treatment for a hyperglycemic episode. Which of the client's actions indicates that he has achieved a cognitive outcome in the management of his new health problem?

The client is able to explain when and why he needs to check his blood sugar.

At the beginning of prenatal care, the goal for the client was to gain 25 lb (11.25 kg) by the end of the pregnancy. At 30 weeks of pregnancy, the client has only gained 1 lb (0.45 kg). Which statement(s) would help the nurse most appropriately interpret these data?

The client is not achieving the goal. The nurse should determine the reasons the client has not been gaining weight.

Which client outcome is a cognitive outcome? Select all that apply.

The client lists the side effects of digoxin (Lanoxin). The client describes how to perform progressive muscle relaxation. The client identifies signs and symptoms of hypoglycemia.

Which client outcome is a psychomotor outcome? Select all that apply.

The client measures capillary blood sugar level. The client catheterizes self, using clean technique.

A nurse caring for a confused client who is not allowed to get out of bed asks the physician for an order for restraints. Which of the following are guidelines for the use of restraints on clients? Select all that apply.

The client's family must be involved in the decision and plan of care. Alternatives to restraints and less restrictive interventions must have been implemented and failed. The benefit gained from using a restraint must outweigh the known risks for that client.

Which client outcome is an example of a physiologic outcome?

The client's pulse oximetry reading is 97% on room air 30 minutes after removal of a nasal cannula.

A nurse is performing eye assessments at a community clinic. Which assessment would the nurse document as normal?

The client's pupils are black, equal in size, and round and smooth.

An resident of a long-term care facility, age 82 years, often tells stories of the role that he played in the Korean War. According to Erikson, which of the following needs may underlie the resident's storytelling?

The need to demonstrate that his life was meaningful and purposeful

Which nurse is using criteria to determine expected standards of performance?

The new graduate nurse consults the policies and procedures of the institution prior to skill implementation.

The newly hired nurse is caring for a client who had a tracheostomy four hours ago. Which action by the nurse would cause the charge nurse to intervene?

The newly hired nurse delegates care of the tracheostomy to a licensed practical/vocational nurse (LPN/LVN).

Which of the following best summarizes the evaluating step of the nursing process? a) The nurse and client identify nursing diagnoses and appropriate interventions. b) The nurse completes a health assessment to establish a database. c) The nurse and client measure achievement of planned outcomes of care. d) The client and family have met health care goals and no longer need care.

The nurse and client measure achievement of planned outcomes of care.

Which nursing action reflects evaluation?

The nurse assesses the client's response to pain medication.

Which nursing action reflects evaluation? a) The nurse sets an anxiety level of 3 or less with the client. b) The nurse identifies that the client has wound drainage. c) The nurse performs a colostomy irrigation. d) The nurse assesses the client's response to pain medication.

The nurse assesses the client's response to pain medication.

Which of the following nursing actions reflects evaluation?

The nurse assesses urine output following administration of a diuretic.

A nurse is following the rules recommended by the Institute of Medicine's Committee on Quality of Health Care in America to help redesign and improve patient care. Which nursing actions are based on these rules? (Select all that apply.)

The nurse bases patient care on established nursing needs and values. The nurse bases care on evidence-based decision making. The nurse promotes shared knowledge and the free flow of information. The nurse acknowledges that continuous decrease in waste improves patient care.

A physician orders an oropharyngeal airway to be inserted into an unconscious patient. Which of the following accurately describes the use of this device?

The nurse can insert this device at the bedside with little to no trauma to the unconscious patient.

A nurse is evaluating nursing care and patient outcomes by using a retrospective evaluation. Which action would the nurse perform in this approach?

The nurse devises a post-discharge questionnaire to evaluate patient satisfaction.

A nurse manager attempts to achieve performance improvement in the emergency department of a busy inner-city hospital. Which nursing actions follow Haase and Miller's recommended steps in performance improvement? Select all that apply.

The nurse discovers that there is a problem with the triage system that is in place in the emergency department. The nurse calls a meeting of the emergency department interdisciplinary team to affect change in the triage process. The nurse organizes a task force to implement change in the triage process of a busy emergency department. The nurse meets with the emergency department staff to assess changes made to the triage process.

A nurse evaluates clients prior to discharge from a hospital setting. Which action is the most important act of evaluation performed by the nurse?

The nurse evaluates the client's goal/outcome achievement.

The nurse is caring for a 35-year-old female patient who reports occasional panic attacks, fatigue, and amenorrhea. What can the nurse infer about this patient? The patient has acute stress. The patient has chronic stress. The patient has a normal response to stress. The patient has a fight-or-flight reaction to stress.

The patient has chronic stress.

A nurse who collected and organized data during a patient history realizes that there is not enough information to plan interventions. Which of following would be the best remedy to prevent this from happening in the future? A. The nurse should practice interviewing strategies B. The nurse should modify data collection tool C. The nurse should determine specific purpose of data collection D. The nurse should review and practice communication techniques

The nurse should practice interviewing strategies

A nurse is completing an intake assessment. The nurse notes that an older adult male client appears to have bruises in varying stages of healing. Which action by the nurse indicates an understanding of her responsibilities?

The nurse should question the client about the source of the bruises.

A nurse is preparing to file a safety event report after a client experienced a fall. Which statement is correct regarding the filing of a safety event report?

The nurse should record the incident in the client's medical record and fill out a safety event report separately.

A nurse is caring for a client in the immediate postoperative period and discovers there are factors that are affecting the attainment of client goals. Which of the following is true of factors that influence client responses and outcome achievement? Select all that apply.

The nurse will draw on positive factors to deal with other variables in the future. A client's strong motivation to learn appropriate health behaviors is an example of a positive factor. The nurse should understand which factors are helpful to attaining outcome attainment and manipulate them to achieve goals.

What is the rationale for placing a writing board in the room of a patient who has had surgery to insert a tracheostomy tube?

The patient is not able to speak.

Which of the following best describes acute illness? a) The leading health problem in the world. b) A medical condition that is life threatening and requires surgery. c) The rapid onset of symptoms lasting a relatively short time. d) A condition that causes a permanent change and a long period of care.

The rapid onset of symptoms lasting a relatively short time.

A nurse was injured when a client with Alzheimer's disease struck the nurse on the side of the head during a transfer. The nurse has completed an incident report. Which statement about an incident report is most accurate?

The report provides a detailed and objective account of the circumstances before, during, and after the event.

In light of the failure of alternatives, a nurse has been forced to physically restrain an agitated client. Which of the following actions should the nurse perform when applying and maintaining the restraints?

Tie the client's hand restraint to the bed frame rather than the side rail.

A nurse performs an assessment of a patient in a long-term care facility and records baseline data. The nurse reassesses the patients a month later and makes revisions in the plan of care. What type of assessment is the second assessment? A. Comprehensive B. focused C. Time-lapsed D. emergency

Time-Lapsed

A patient scheduled for surgery seems to be anxious. The nurse teaches the patient to practice deep breathing exercises. What is the most appropriate reason for teaching deep breathing exercises to the patient? To help the patient prevent postoperative complications. To prevent preoperative risk of respiratory complications. To dampen the cognitive process involved in anxiety reactions. To promote lung expansion during administration of anesthesia.

To dampen the cognitive process involved in anxiety reactions.

Primary health promotion is an integral part of health care. What is the purpose of primary health promotion?

To promote health and prevent disease or injury.

Which factor is related to the highest proportion of falls in long-term care settings?

Toileting

The nurse has observed that a client's food intake has diminished in recent days. What intervention should the nurse perform in order to stimulate the client's appetite?

Try to ensure that the client's food is attractive and sufficiently warm.

A nurse is caring for a client with a history of cardiac and vascular disease. Which of the following fats should the nurse allow in the client's diet for his condition?

Unsaturated fats

A client's diagnosis of breast cancer necessitates a bilateral mastectomy and breast reconstruction with tissue expanders. The nurse recognizes that the client's surgery will have a significant impact on her activities of daily living (ADLs) during her period of recovery. When should discharge planning to address ADLs begin for this client?

Upon her admission to the hospital

A nurse is caring for an elderly client. What strategy should the nurse include in order to facilitate effective communication?

Use active listening during communication.

The nurse is addressing primary prevention with a group of college students. Which is an example of a primary prevention measure? a) annual pap smears for the female students b) testicular examinations for the male students c) HIV screening d) use of condoms and safer sex practices

Use of condoms and safer sex practices

There is a fire in the neurology unit of a healthcare facility. Which of the following would be most appropriate to avoid in this situation?

Use of elevators

The nurse is following the Joint Commission's national patient safety goals when giving medications. Based on these goals, how can the nurse improve the accuracy of patient identification?

Use two patient identifiers (neither to be the room number).

The nurse is preparing to assess the client's vital signs. The client just had their morning coffee. What is the appropriate nursing intervention?

Wait 30 minutes, then assess oral temperature.

Which actions should a nurse perform when inserting an oropharyngeal airway?

Wash hands and put on PPE, as indicated. Rotate the airway 180 degrees as it passes the uvula. Remove airway for a brief period every 4 hours or according to facility policy.

The nurse is teaching a nursing student about proper latex glove use. Which teaching will the nurse include?

Wash hands thoroughly after removing gloves with a pH balanced soap.

Which of the following factors is related to developmental changes in bowel habits for elderly clients? a) Increase in dietary fiber can decrease peristalsis b) Weakened pelvic muscles lead to constipation c) The elderly should peel fruits before eating d) Milk products cause constipation in lactose intolerance

Weakened pelvic muscles lead to constipation

The nurse is assessing a 4-year-old child who has multiple bee stings. Which assessment finding would require immediate action by the nurse?

Wheezing on auscultation

A nurse is caring for a client with influenza who requires an external source of oxygen in order to breathe efficiently. In which of the following situations is oxygen humidified?

When more than 3 L/min of oxygen is administered for an extended period

Which phase of the nurse-client relationship involves the mutual planning of care and putting the plan into action?

Working

A dialysis nurse is educating a client on caring for the dialysis access that was inserted into the client's right arm. The nurse assesses the client's fears and concerns related to dialysis, the dialysis access, and care of the access. This information is taught over several sessions during the course of the client's hospitalization. Which phase of the relationship is best described in this scenario?

Working Phase

A woman anticipating her wedding ceremony in a week comes to the Urgent Care Clinic with a headache that has been ongoing for three days duration as well as a pounding feeling in her chest. She is concerned that she may be having a stroke or a cerebral vascular accident (CVA). All medical tests are negative. The woman wonders what is happening to her because she feels she is unable to function. What is the best response to her question? You are having difficulty coping with your upcoming wedding. You are experiencing distress because of your upcoming wedding. You are experiencing eustress because of your upcoming wedding. You are being warned by your body that you may have hypertension.

You are having difficulty coping with your upcoming wedding.

What age group is most vulnerable to toxic fumes or asphyxiation?

Young children

A nurse is volunteering at a day camp. A child is stung by a bee and develops wheezing in the upper airways. The child is experiencing:

a bronchospasm.

In which client would the nurse assess for a depressed respiratory system?

a client taking opioids for cancer pain

It is very important to assess for the quality of someone's respirations as well as describe what is heard with auscultation. Which describes stridor?

a harsh, inspiratory sound that may be compared to crowing It is very important to assess for the quality of someone's respirations as well as describe what is heard with auscultation. Which describes stridor?

A large university hospital has commissioned a multidisciplinary group to review client records following discharge in order to evaluate client outcomes and the character and quality of nursing care that clients receive. What type of evauation process will take place?

a nursing audit.

Which of the following patients should have their vital signs monitored at least every 4 hours? A) a patient in a critical care unit B) a patient hospitalized for high blood pressure C) a resident in a long-term care facility D) a long-term care resident on Medicare A

a patient hospitalized for high blood pressure

Mrs. Miller is a 60-year-old woman status post a hip replacement. She has had multiple complications following surgery including a skin infection and a blood clot. As a result, she has been a client on the unit for 6 weeks. The nurse has just returned from vacation and this is her first day caring for Mrs. Miller. A colleagues looks at the nurse and describes Mrs. Miller as "quite difficult to deal with." The nurse knows that all of the following can contribute to difficult behaviors except:

a quiet room

The definition of chronic conditions can be complex. Which factors would a nurse expect to be included in the definition of chronic conditions? (Select all that apply.) a) Diseases that have a prolonged course b) Diseases that have a short course c) Diseases that do not resolve spontaneously d) Diseases that resolve spontaneously e) Diseases where complete cures are rare

a) Diseases that have a prolonged course c) Diseases that do not resolve spontaneously e) Diseases where complete cures are rare

Which of the following activities would the nurse consider to be lifestyle risk factors for the development of health problems? Select all that apply. a) avoiding exercise b) family history of hypertension c) living in an area with high smog levels d) participating in extreme sports e) maintaining a stressful job

a) avoiding exercise d) participating in extreme sports e) maintaining a stressful job

Stress can be attributed to stimulation of the hypothalamus-pituitary-adrenal cortex, and over the long term it can result in a. Insulin resistance b. Digestive problems c. Chronic muscle tension d. A high resting heart rate

a. Insulin resistance and obesity are considered long-term sequelae of the high blood glucose levels incurred when the body responds to stress. pp. 158, 160, Figure 10.2, Table 10.1

Self-help groups are useful for reducing stress because they provide the individual with the stress mediator identified as a. Social support b. Cultural support c. Life satisfaction d. Cognitive reframing

a. Self-help groups often provide a high level of social support. Members meet and are encouraged and sustained by others who share the same problem. p. 161

Which of the following statements by a new graduate nurse should be corrected by an experienced nurse? a. "Most older patients are ill and disabled. That's why we care for so many of them in the hospital." b. "Older adults are many times still interested in sexual relations." c. "Patients over age 65 are still lifelong learners." d. "Many older adult patients remain independent enough to live alone."

a. "Most older patients are ill and disabled. That's why we care for so many of them in the hospital." Although many experience chronic conditions or have at least one disability that limits their performance of activities of daily living, in 2004, 37.4% of noninstitutionalized older adults assessed their health as excellent or very good. Older adults do report continued enjoyment of sexual relationships. Although changes in vision or hearing and reduced energy and endurance sometimes affect the process of learning, older adults are lifelong learners. Most older adults live in noninstitutional settings with family members or alone.

The nurse correctly describes psychosocial theories on aging as theories that a. Describe role changes in behaviors in older adults. b. Emphasize that all adults age in similar ways. c. Stress the need for the aging to discontinue activities as they age. d. Describe behavior patterns for all aging adults as unpredictable.

a. Describe role changes in behaviors in older adults. Psychosocial theories of aging explain changes in behaviors, roles, and relationships that come with aging. Although some theories generalize about aging, biologically and psychosocially each individual ages uniquely. The activity theory considers the continuation of activities performed during middle age as necessary for successful aging. The continuity theory states that personality remains the same and behavior becomes more predictable as people age.

Several theories on aging have been put forth, and the nurse should use these theories to a. Guide nursing care. b. Explain the stochastic view of genetically programmed physiological changes. c. Select one theory to guide nursing care for all geriatric patients. d. Understand the nonstochastic views of aging as the result of cellular damage.

a. Guide nursing care. Although theories on aging are in various stages of development and have limitations, the nurse should use them to increase understanding of the phenomena affecting the health and well-being of older adults and to guide nursing care. Stochastic theories view aging as the result of random cellular damage occurring over time. No one single universally accepted theory predicts and explains the complexities of the aging process. Nonstochastic theories view aging as the result of genetically programmed physiological mechanisms within the body.

16. A patient with gradual, progressive cognitive impairment (dementia) is admitted to the nursing unit after hip replacement surgery. Which of the following is a nursing care principle for care of cognitively impaired older adults? a. Maintain physical health. b. Evaluate the patient's manifestations of standard symptoms. c. Assist patient with all ADLs. d. Isolate patients to protect others.

a. Maintain physical health. The nurse works to monitor and maintain physical health. The nurse should also assess the person's unique manifestations of the disease as it progresses while facilitating independent performance of activities of daily living (ADLs). Social interaction based on the patient's abilities is to be promoted.

One of the greatest challenges for the nurse caring for older adults is ensuring safe medication use. One way to reduce the risks associated with medication usage is to a. Periodically review the patient's list of medications. b. Inform the patient that polypharmacy is to be avoided at all cost. c. Be aware that medication is absorbed the same way regardless of patient age. d. Focus only on prescribed medications.

a. Periodically review the patient's list of medications. Periodic and thorough review of all medications is important to restrict the number of medications used to the fewest necessary to ensure the greatest therapeutic benefit with the least amount of harm. Although polypharmacy reflects inappropriate prescribing, the concurrent use of multiple medications is necessary in situations where an older adult has multiple acute and chronic conditions. Older adults are at risk for adverse drug effects because of age-related changes in the absorption, distribution, metabolism, and excretion of drugs. Work collaboratively with the older adult to ensure safe and appropriate use of all medications—both prescribed medications and over-the-counter medications.

An older adult patient has developed acute confusion. The patient has been on tranquilizers for the past week. The patient's vital signs are normal. What should the nurse do? a. Take into account age-related changes in body systems that affect pharmacokinetic activity. b. Increase the dose of tranquilizer if the cause of the confusion is an infection. c. Note when the confusion occurs and medicate before that time. d. Restrict telephone usage to prevent further confusion.

a. Take into account age-related changes in body systems that affect pharmacokinetic activity. Sedatives and tranquilizers sometimes prescribed for acutely confused older adults sometimes cause or exacerbate confusion. Carefully administer drugs used to manage confused behaviors, taking into account age-related changes in body systems that affect pharmacokinetic activity. When confusion has a physiological cause (such as an infection), specifically treat that cause, rather than the confused behavior. When confusion varies by time of day or is related to environmental factors, nonpharmacological measures such as making the environment more meaningful, providing adequate light, etc., should be used. Making telephone calls to friends or family members allows older adults to hear reassuring voices, which may be beneficial.

Which of these findings, if identified in a patient on a gerontological unit, would be most surprising to a culturally sensitive nurse? a. The older person not being functionally independent b. Preferences in food, music, and religion c. Use of conventions of the handshake, silence, and eye contact d. Personal health practices and spiritual resources

a. The older person not being functionally independent Most older people remain functionally independent despite the increasing prevalence of chronic disease. Examples of culturally competent nursing approaches to older adults include respect for preferences in food, music, and religion; appropriate use of conventions of the handshake, silence, and eye contact; use of interpreters; use of physical assessment norms appropriate for the ethnic group; and asking about personal health practices, family customs, lifestyle preferences, and spiritual resources.

An evening shift nurse is caring for a client scheduled for a colon resection in the morning. The client tells the nurse she is afraid of waking up during surgery. The best response by the nurse is to:

ask the client why she thinks she will wake up during surgery.

A graduate nurse has been working on a telemetry unit for 6 months. The nurse arrives at work in the morning and overhears a night shift nurse talking about the graduate nurse. The night shift nurse is heard saying, "That new nurse is only here to meet a doctor and get married." The best response by the new nurse would be to:

ask to speak to the night shift nurse in private and explain how the comment made her feel.

A client is administered an anxiolytic. Which nursing action demonstrates the nurse evaluating the client?

asking if the client feels less anxious 30 minutes after administering the medicine

A nurse's neighbor tells her, "I have a high temperature, feel awful, and I am not going to work." What stage of illness behavior is the neighbor exhibiting? a) experiencing symptoms b) assuming the sick role c) assuming a dependent role d) achieving recovery and rehabilitation

assuming the sick role

A patient with a lump in her breast calls her primary care physician to schedule an appointment for an evaluation. Based upon the patient's actions, what stage of illness is the patient demonstrating? a) experiencing the symptoms b) assuming the sick role c) achieving recovery and rehabilitation d) assuming the dependent role

assuming the sick role

The nurse is caring for a postoperative adult client who has developed pneumonia. The nurse should assess the client frequently for symptoms of:

atelectasis

An 17-year-old is brought to the emergency department with a head injury. The nurse knows that adolescents are vulnerable to injuries related to:

automobile accidents.

A client with chronic obstructive pulmonary disease requires low-flow oxygen. How will the oxygen be administered? Select all that apply.

b) Nasal cannula c) Simple oxygen mask e) Partial rebreather mask

The first stage of the general adaptation syndrome can be characterized as a. Eustress b. Adaptive c. Maladaptive d. Psychological

b. The initial adaptive response of the general adaptation syndrome prepares the individual to fight or flee in the face of acute stress.

An outcome for an older adult patient living alone is to be free from falls. Which of these statements by a patient indicates that teaching on safety concerns has been effective? a. "I'll leave my throw rugs in place so that my feet won't touch the cold tile." b. "I'll take my time getting up from the bed or chair." c. "I should wear my favorite smooth bottom socks to protect my feet when walking around." d. "I will have my son dim the lighting outside to decrease the glare in my eyes."

b. "I'll take my time getting up from the bed or chair." Older adults taking medications with adverse effects such as postural hypotension, dizziness, or sedation need to be aware of these potential effects and to take precautions such as changing position slowly or ambulating with assistance if unsteady. Household items that are easy to trip over, such as throw rugs, are a risk factor for falls. Other risk factors include wearing shoes in poor repair or slippery soles. Impaired vision and poor lighting are other risk factors.

Which of these assessments of an older adult, who has a urinary tract infection, requires an immediate nursing intervention? a. Presbycusis b. Confusion c. Death of a spouse 3 months ago d. Temperature of 97.6° F

b. Confusion Confusion is a common manifestation in older adults with urinary tract infection; however, the cause requires further assessment. There may be another reason for the confusion. Confusion is not a normal finding in the older adult, even though it is commonly seen with concurrent infections. Difficulty hearing, presbycusis, is an expected finding in an older adult. Coping with the death of a spouse is a psychosocial concern to be addressed after the acute physiological concern in this case. Older adults tent to have lower temperatures, so the nurse needs to assess for slight elevations. A temperature of 97.6° F is within normal limits.

As the aging population in the United States increases, the nurse knows that the a. Baby boomer generation accounts for a very small percentage of this group. b. Extension of the average life span has also increased. c. Population segment over age 85 is decreasing. d. Diversity of this age group will certainly decrease.

b. Extension of the average life span has also increased. According to estimates, the number of older adults will increase to 72.1 million by 2030. Part of that increase is due to extension of the average life span. Two other factors that contribute to the projected increase in the number of older adults are the aging of the baby boom generation and the growth of the population segment over age 85. The baby boomers are the large group of adults born between 1946 and 1964.The diversity of the group over age 65 will also possibly increase.

As a patient ages, the nursing plan of care a. Should be standardized because all geriatric patients have the same needs. b. Needs to be individualized to the patient's unique needs. c. Should be based on chronological age alone. d. Focuses on the disabilities that all aging persons face.

b. Needs to be individualized to the patient's unique needs. Every older adult is unique, and the nurse needs to approach each one as a unique individual. The nursing care of older adults poses special challenges because of great variation in their physiological, cognitive, and psychosocial health. Nurses need to take into account the cultural, ethnic, and racial diversity represented by these numbers (not just age) as they care for older adults from these groups. Aging does not inevitably lead to disability and dependence.

An 80-year-old male is brought to the emergency department with an exacerbation of chronic obstructive pulmonary disease (COPD). He states that he quit smoking 30 years ago, so it can't be COPD. He argues, "It's just these colds I've been getting. They're just getting worse and worse." The nurse understands that a. These symptoms are more associated with normal aging than with disease. b. Older adults do not have to alter physical activity because of physical changes. c. The patient's age will require adjustment of lifestyle to one of inactivity. d. Older adults usually are aware and accepting of the aging process.

b. Older adults do not have to alter physical activity because of physical changes. Older adults face the necessity of adjustment to the physical changes that accompany aging. As body systems age, changes in appearance and functioning occur. These changes are not associated with a disease but are normal changes. The presence of disease sometimes alters the timing of the changes or their impact on daily life. Acceptance of personal aging does not mean retreat into inactivity, but it does require a realistic review of strengths and limitations. Some older adults find it difficult to accept that they are aging.

During a routine prenatal care visit, a pregnant woman in her last trimester of pregnancy reports that she has occasional shortness of breath. The nurse instructs her that:

breathing becomes increasingly difficult as the diaphragm is displaced.

While assessing vital signs of a patient with a head injury and increased intracranial pressure (IICP), a nurse notes that the patient's respiratory rate is 8 breaths/min. How will the nurse interpret this finding? A) bradypnea is uncommon in patient with IICP B) IICP most commonly results in tachypnea C) bradypnea is a response to IICP D) this is a normal respiratory rate

bradypnea is a response to IICP

A patient is extremely depressed after ending a relationship with a partner. The nurse is teaching cognitive reframing techniques to this patient as a means to counteract depression. Which response given by the patient indicates effective teaching by the nurse? a. "'My partner will realize that I was not wrong." b. "I was destined to face this situation in my life." c. "I have many friends who love and care for me." d. "I will never think about my partner again in my life."

c. The nurse should teach cognitive reframing to patients who are depressed and suffer from self-blame. It helps the patients develop positive insight towards life and real-life situations. In this regard, the patient's response, "I have many friends who love and care for me," is appropriate because it is a positive statement. The response, "I will never think about my partner again in my life," is not appropriate. It indicates that the patient is not accepting the reality and avoids thoughts related to the incident. The response, "My destiny is responsible for the breakup," is not appropriate either. It indicates that the patient is compromising with the situation. The response, "My partner will realize that I was not wrong," is not appropriate. The patient is reflecting anger and distress in this response. p. 167

An adult patient admitted with anxiety states, "My counselor keeps talking about my stressors. What exactly are stressors?" How does the nurse best respond to the patient? a. "Stressors are complicated neurostimuli that cause mental illness." b. "Instead of focusing on what stressors are, let's explore your coping skills." c. "Stressors are events that threaten your current functioning and require you to adapt." d. "It's best if you ask questions like that of your health care provider for a complete answer."

c. "Stressors are events that threaten your current functioning and require you to adapt." The statement "Stressors are events that threaten your current functioning and require you to adapt" is the best explanation of stressors. Stressors are not complicated neurostimuli; telling the patient to address these questions to the health care provider fails to educate the patient, which is the nurse's responsibility. Exploring coping skills would be a good intervention at a later time but does not address the patient's question and changes the subject.

A 70-year-old patient who suffers from worsening dementia is no longer able to live alone. When discussing health care services and possible long-term living arrangements with the patient's only son, what should the nurse suggest? a. An apartment setting with neighbors close by b. Having the patient utilize weekly home health visits c. A nursing center because home care is no longer safe d. That placement is irrelevant because the patient is retreating to a place of inactivity

c. A nursing center because home care is no longer safe Some family caregivers consider nursing center placement when in-home care becomes increasingly difficult, or when convalescence from hospitalization requires more assistance than the family is able to provide. An apartment setting and the use of home health visits are not appropriate because some older adults deny functional decline and refuse to ask for assistance with tasks that place their safety at great risk. Others avoid activities designed to benefit older adults such as senior health promotion activities (such as some health visits), and thus do not receive the benefits that these programs offer. Acceptance of personal aging does not mean a retreat into inactivity, but it does require a realistic review of strengths and limitations.

When comparing developmental tasks of middle-aged persons versus older adults, what should the nurse infer? a. Learning to cope with loss is most common during the middle adult years. b. After age 65, most older adults age both biologically and psychologically the same way. c. All older adults will need nursing assistance to deal with loss. d. Older adults fear and resent retirement as a disruption of their lifestyle.

c. All older adults will need nursing assistance to deal with loss. Some older adults deny their own aging in ways that are potentially problematic. For example, some older adults deny functional declines and refuse to ask for assistance with tasks that place their safety at great risk. The need to cope with loss is much greater in the older adult population. Most older adults cope with the death of a spouse. Some must cope with the death of adult children and grandchildren. All experience the death of friends. The ways that older adults adjust to the changes of aging are highly individualized. Many older adults welcome retirement as a time to pursue new interests and hobbies, participate in volunteer activities, continue their education, or start a new business career.

Which teaching strategy is best to utilize with older adult patients? a. Provide several topics of discussion at once to promote independence and making choices. b. Avoid uncomfortable silences after questions by helping patients complete their statements. c. Ask patients to recall past experiences that correspond with their interests. d. Speak in a high pitch to help patients hear better.

c. Ask patients to recall past experiences that correspond with their interests. Teaching strategies include the use of past experiences to connect new learning with previous knowledge, focusing on a single topic to help the patient concentrate, giving the patient enough time in which to respond because older adults' reaction times are longer than those of younger persons, and keeping the tone of voice low; older adults are able to hear low sounds better than high-frequency sounds.

A male older adult patient expresses his concern and anxiety about decreased penile firmness during erection. What is the nurse's best response? a. Explain that over time, his libido will decrease, as will the frequency of sexual activity. b. Tell the patient to double his antidepressant medication to increase his libido. c. Tell the patient that this change is expected in aging adults. d. Tell the patient that touching should be avoided unless intercourse is planned.

c. Tell the patient that this change is expected in aging adults. Decreased firmness during erection is an expected change in aging adults. Libido does not necessarily decrease as one ages. Many older adults use prescription medications that depress sexual activity such as antihypertensives, antidepressants, sedatives, or hypnotics. Touch complements traditional sexual methods or serves as an alternative sexual expression when physical intercourse is not desired or possible.

A nurse is caring for a client with pneumonia. The client's oxygen saturation is below normal. What abnormal respiratory process does this demonstrate?

changes in the alveolar-capillary membrane and diffusion

Peripheral cyanosis and clubbing of the nails are symptoms of:

chronic hypoxia.

When the nurse prepares to discharge a client, and subsequently evaluate the effectiveness of the patient care, the nurse should determine whether the:

client's goals have been achieved.

Which of the following groups of terms best defines assessing in the nursing process? A. Problem focused, time lapsed, emergency based B. design a plan of care, implement nursing interventions C. collection, validation, communication of patient data D. nurses focused, establishing nursing goals

collection, validation, communication of patient data

Of the following data, what type would be collected during a physical assessments? A. color, moisture, and temperature of the skin B. type, amount, and duration of pain C. food eaten that cause nausea D. specific allergies resulting in itching

color, moisture, and temperature of the skin

A Hispanic woman has been in an abusive relationship for 3 years. Which comment by this patient is most likely? a. "I need help with preparing an escape plan." b. "I feel so depressed about the failure of my marriage." c. "He is not a good man. His father and brother are also abusive." d. "I have very bad headaches that make it hard for me to care for my children."

d. The patient is experiencing severe stress but expressing it somatically. The overwhelming majority of Asians, Africans, and Central Americans tend to express distress in somatic terms and actually experience it physically. In the correct answer, the patient has expressed her distress about the abuse with complaints of headaches. Although Western European and North American cultures tend to subscribe to a psychophysiologic view of stress and somatic distress, this is not the dominant view in other cultures. The incorrect responses depict emotional recognition of stress associated with the abuse. p. 162

Which of these patient statements is the most reliable indicator that an older adult has the correct understanding of health promotion activities? a. "I need to increase my fat intake and limit protein." b. "I should discontinue my fitness club membership for safety reasons." c. "I'm up to date on my immunizations, but at my age, I don't need the tetanus vaccine." d. "I still keep my dentist appointments even though I have partials now."

d. "I still keep my dentist appointments even though I have partials now." General preventive measures for the nurse to recommend to older adults include keeping periodic dental appointments to promote good oral hygiene, eating a low-fat, well-balanced diet, exercising regularly, and maintaining immunizations for influenza, pneumococcal pneumonia, and tetanus.

What is the best suggestion a nurse could make to a family requesting help in selecting a local nursing center? a. Suggest choosing a nursing center that is as sanitary as possible. The closer the center is to hospital standards, the better. b. Have family members evaluate nursing home staff according to their ability to get tasks done efficiently. c. Make sure that nursing home staff members get patients out of bed every day for the entire day. d. Explain that it is probably best for the family to visit the center and inspect it personally.

d. Explain that it is probably best for the family to visit the center and inspect it personally. An important step in the process of selecting a nursing home is to visit the nursing home. The nursing home should not feel like a hospital. It is a home, a place where people live. Members of the nursing home staff should focus on the person, not the task. Residents should be out of bed and dressed according to their preferences, not staff preferences.

An older adult patient in no acute distress reports being less able to taste and smell. What is the nurse's best response to this information? a. Notify the physician immediately to rule out cranial nerve damage. b. Perform testing on the vestibulocochlear nerve and a hearing test. c. Schedule the patient for an appointment at a smell and taste disorders clinic. d. Explain to the patient that diminished senses are normal findings.

d. Explain to the patient that diminished senses are normal findings. Diminished taste and smell senses are common findings in older adults. Scheduling an appointment at a smell and taste disorders clinic, testing the vestibulocochlear nerve, or an attempt to rule out cranial nerve damage is unnecessary at this time per the information provided.

To promote physical well-being and socialization in an older adult, what should the nurse realize? a. Social isolationism is always a chosen behavior. b. Body image plays no role in decision making by the older adult. c. No community resources are focused on the older adult. d. Older adults may have a functional purpose in social arenas.

d. Older adults may have a functional purpose in social arenas. Social service agencies in most communities welcome older adults as volunteers and provide the opportunity for older adults to serve while meeting their socialization or other needs. Although some older adults choose isolation or a lifelong pattern of reduced interaction with others, other older adults do not choose isolation but are vulnerable to its consequences. Some older adults withdraw from social interaction because of feelings of rejection. These older adults see themselves as unattractive and rejected because of changes in their personal appearance due to normal aging changes or because of body image changes. Many communities have outreach programs designed to make contact with isolated older adults.

An older patient has fallen and broken his hip. As a consequence, the patient's family is concerned about his ability to care for himself, especially during his convalescence. What should the nurse do? a. Stress that older patients usually ask for help when needed. b. Inform the family that placement in a nursing center is a permanent solution. c. Tell the family to enroll the patient in a ceramics class to maintain his quality of life. d. Provide information and answer questions as family members make choices among care options.

d. Provide information and answer questions as family members make choices among care options. Nurses assist older adults and their families by providing information and answering questions as they make choices among care options. Some older adults deny functional declines and refuse to ask for assistance with tasks that place their safety at great risk. The decision to enter a nursing center is never final, and a nursing center resident sometimes is discharged to home or to another less-acute residence. What defines quality of life varies from person to person. Nurses must listen to what the older adult considers to be most important rather than making assumptions about the individual's priorities.

A 72-year-old woman was recently widowed. She worked as a teller at a bank for 40 years and has been retired for the past 5 years. She never learned how to drive. She lives in a rural area that does not have public transportation. Which of the following psychosocial changes does the nurse focus on as a priority? a. Sexuality b. Housing and environment c. Retirement d. Social isolation

d. Social isolation The highest priority at this time is the potential for social isolation. This woman does not know how to drive and lives in a rural community that does not have public transportation. All of these factors contribute to her social isolation. Other possible changes she may be going through right now include sexuality related to her advanced age and recent death of her spouse; however, this is not the priority at this time. She has been retired for 5 years, so this is also not an immediate need. She may eventually experience needs related to housing and environment, but the data do not support this as an issue at this time.

A nurse designs a care plan to improve walking mobility in an older adult client. When encouraged to implement the new strategies for ambulation the client refuses to try and tells the nurse, "I find it easier to use a wheelchair." What action by the nurse may have led to failure to meet the outcome?

developing the plan without client input

Oxygen and carbon dioxide move between the alveoli and the blood by:

diffusion

A client with Crohn's disease in remission is admitted to the nursing unit for follow-up care. The remission state is characterized by: a) permanent relief from the signs and symptoms. b) disappearance of signs and symptoms associated with the disease. c) periodic occurrence in clients with long-standing diseases. d) reactivation of the disease and presence of symptoms.

disappearance of signs and symptoms associated with the disease.

After the nursing plan of care has been developed, the nurse knows that:

each encounter with the client is an opportunity to reassess and revise the plan of care if necessary.

A client with no prior history of respiratory illness has been admitted to a postoperative unit following foot surgery. What intervention should the nurse prioritize in an effort to prevent postoperative pneumonia and atelectasis during this time of reduced mobility following surgery?

educating the client on the use of incentive spirometry

A nurse is preparing to provide discharge instructions to a postpartum client regarding infant care. Before beginning the education session, the nurse should:

eliminate as many distractions as possible.

Which term describes a nurse who is sensitive to the client's feelings, but remains objective enough to help the client achieve positive outcomes?

empathetic

Which of the following diseases may result in decreased lung compliance?

emphysema

The nurse in a Burn Intensive Care Unit (BICU) is caring for a 3-year-old boy who was burned with scalding hot water. He has burns covering 75 percent of his body. His condition is critical but stable. At 1000, the nurse reassesses the client and finds that he is agitated and pulling at his endotracheal tube. What would be the nurse's priority?

ensuring that the endotracheal tube is secure

A nurse is gathering equipment needed for a basic physical assessment. Which supplies will be required? (Select all that apply.)

gloves, examination gown, cloth or paper drapes, scale, stethoscope, sphygmomanometer, thermometer, pen light or flashlight, tongue blade, assessment form, and pen.

A client with a recently fractured left femur has been reluctant to comply with his physical therapy for fear of the pain associated with movement. A goal for this client is to attend therapy treatments 3 times each day. The nurse is evaluating the goal for this client. The client states, "I don't like therapy; it hurts, but I have been going twice a day." The client chart has an entry from the last shift nurse stating the client went to therapy 2 times with encouragement. The nurse evaluates the goal as:

goal partially met.

A nurse educator uses models of health and illness when teaching. Which model of health and illness places high-level health and death on opposite ends of a graduated scale?

health-illness continuum

When chronic illnesses and disabilities are present, individuals benefit most from activities that: a) preserve their social interactions. b) help them eat well. c) help them maintain independence. d) accomplish financial stability.

help them maintain independence.

A nurse is providing care to a 3-year-old child admitted with a diagnosis of infectious diarrhea. The nurse needs to insert an intravenous catheter in order to administer prescribed intravenous fluids. In an attempt to foster communication, the nurse should:

involve the child's stuffed animal in the educational session.

Which topic should a public health nurse emphasize when educating older adults on reducing their risk of poisoning?

keeping medications in clearly labeled containers

During an annual performance review with an employee, the nurse manager does not maintain eye contact and seems concerned about the time and the next appointment. This use of communication is considered

nonverbal

A student is reading the medical record of an assigned patient and notes the patient has been afebrile for the past 12 hours. What does the term "afebrile" indicate? A) normal body temperature B) decreased body temperature C) increased body temperature D) fluctuating body temperature

normal body temperature

The primary purpose for evaluating data about a client's care according to a functional health approach is to:

revise or modify the patient care plan.

A nurse is collecting data from a home care patient. In addition to information about the patient's health status, what is another observation the nurse should make? A. Number of rooms in the house B. safety of the immediate environment C. frequency of home visits to be made D. Friendliness of the patient and family

safety of the immediate environment

A nurse is at the end of a busy shift on a medical-surgical unit. The nurse enters a room to empty the client's urinary catheter and the client says, "I feel like you ignored me today." In response to the statement, the nurse should:

sit at the bedside and allow the client to explain the statement.

A nurse is preparing to conduct a health history for a patient who is confined to bed. How should be the nurse position herself? A. standing at the end of the bed B. standing at the side of the bed C. sitting at least 6 feet from the bedside D. sitting at a 45-degree angle to the bed

sitting at a 45-degree angle to the bed

The terms "criteria" and "standard" are often used interchangeably but actually have distinct, separate definitions. "The levels of performance accepted by, and expected of, nursing staff or other health team members" is known as:

standards.

The nursing supervisor is evaluating how many clients each of the department nurses has been assigned for the shift. This type of evaluation would be considered:

structure.

The nurse is working in the newborn nursery and observes neonates in various states of health and wellness. The nurse is aware that which of the following factors can affect fetal development? Choose all that apply.

• Age of mother • Prenatal nutrition • Substance abuse by mother

A nurse working in the ICU understands that illnesses affect both the client as well as the family members. What are some things that often cause stress in family members due to a hospitalization of a family member? Select all that apply. a) Alterations in lifestyle b) Alterations in roles c) Increase in social interactions d) Economic problems e) Decrease in social interactions

• Alterations in lifestyle • Alterations in roles • Economic problems • Decrease in social interactions

The nurse is educating a high school health and fitness class about substance abuse. One of the group members asks what happens if a pregnant woman is using drugs. The nurse's best replies include which of the following? Choose all that apply.

• Low birth weight • Premature births • Congenital anomalies • Risk of poor nutrition

A student nurse is assisting the school nurse with a health fair at a middle school. The student nurse assesses the students' height and weight as they file through her station, where she observes that there is a large variation in physical size and emotional maturity. Which of the following factors may affect physical growth of children? Choose all that apply.

• Prenatal nutrition • Physical activity • Heredity

Which of the following is a correctly written nursing intervention? Select all that apply.

• Provide 5 to 6 small meals daily. • Reposition the client from side to side every hour around the clock. • Provide opportunities for the client to express concerns and verbalize feelings.


Related study sets

QI 105: Leading Quality Improvement

View Set

UTILITY AND APPLICATIONS OF APPLIED ECONOMICS (MODULE 2)

View Set

Fahrschule Theorie Grundwissen Fragen ohne Bild

View Set

Chapter Quiz - Life Policy Provisions, Riders, and Options

View Set

9 клас English Career choices @Jobs@

View Set